SMB Week 3 UWorld

Ace your homework & exams now with Quizwiz!

A 36-year-old man comes to the office to discuss a skin rash on his shoulders and upper chest. The patient first noticed the rash 2 months ago while vacationing at a beach resort. The rash has worsened progressively and is associated with mild pruritus, but he has no other associated symptoms. The patient is a road construction worker, and often works in hot, humid areas. Past medical history is notable for seasonal allergies and childhood asthma. He does not use tobacco or alcohol. Skin examination findings are shown in the image below. ID=104 Which of the following is the most likely cause of this patient's condition? Blastomyces dermatitidis Candida albicans Histoplasma capsulatum Malassezia globosa Microsporum canis Rhizopus species Sporothrix schenckii

Pityriasis versicolor (also called tinea versicolor) is characterized by hypopigmented, hyperpigmented, or erythematous macules or patches on the upper body. It is caused by Malassezia species yeasts, and the infection is confined to the stratum corneum of the skin. It is common in areas with hot and humid climates and is often most visible after extensive sun exposure due to tanning of the adjacent skin. Most cases of pityriasis versicolor occur in healthy individuals with no underlying immunologic deficiencies. The diagnosis of pityriasis versicolor is made by potassium hydroxide (KOH) preparation of skin scrapings. Malassezia forms spores and hyphae, producing the characteristic "spaghetti and meatballs" appearance on light microscopy. The hyphae have a short, "cigar-butt" appearance. This condition is treated with topical antifungal agents or selenium-containing shampoo. (Choices A and C) Blastomycosis is characterized by progressive pulmonary infection. Skin involvement is uncommon but can manifest as papular or pustular lesions in exposed areas. Histoplasmosis also most commonly causes pulmonary disease, though it may become disseminated in immunodeficient patients. (Choice B) Intertrigo is characterized by well-defined erythematous plaques with satellite vesicles or pustules in warm, moist skin areas (eg, axillae, groin, skin folds). The infection is usually due to Candida species, and it is most common following antibiotic use or in immunocompromised states (eg, uncontrolled diabetes). (Choice E) Microsporum species are a frequent cause of tinea capitis and tinea corporis. (Choice F) Rhizopus is a major cause of mucormycosis, which is characterized by invasive, necrotic upper and lower respiratory infections in immunocompromised patients. (Choice G) Sporothrix molds are found in decaying horticultural matter and are acquired through breaks in the skin. They cause nodular and ulcerating subcutaneous lesions. Educational objective: Pityriasis versicolor (tinea versicolor) is a superficial skin infection caused by Malassezia species. It causes erythematous, hyper- or hypopigmented macules and patches. Malassezia forms spores and hyphae, producing the characteristic "spaghetti and meatballs" appearance on KOH preparation light microscopy.

A 43-year-old woman comes to the emergency department due to low-grade fevers and malaise. She has a history of lung transplantation due to cystic fibrosis and had 2 episodes of cytomegalovirus (CMV) viremia over the past few months, which were treated with ganciclovir. Temperature is 37.6 C (99.7 F). Physical examination is unrevealing. Chest x-ray and urinalysis are negative for infection. Polymerase chain reaction testing for CMV in blood shows elevated CMV levels consistent with CMV viremia. Given the recurrent episodes of viremia, genotype analysis is performed and demonstrates ganciclovir-resistant CMV. The patient is started on an alternate intravenous antiviral agent. She develops hypocalcemia and hypomagnesemia. Which of the following agents is the most likely cause of these side effects? Acyclovir Cidofovir Foscarnet Lamivudine Sofosbuvir Valganciclovir

Foscarnet is a pyrophosphate analog that is sometimes used for ganciclovir-resistant cytomegalovirus (CMV) infections. Foscarnet can chelate calcium. Moreover, foscarnet-induced renal wasting of magnesium may lead to hypomagnesemia and a reduction in the release of parathyroid hormone, which contributes to the hypocalcemic state. Both hypocalcemia and hypomagnesemia can promote seizures. (Choice A) Major toxicities of acyclovir (used in some herpes simplex virus infections) include crystal nephropathy and neurotoxicity that manifests as delirium and/or tremor. (Choice B) Cidofovir, used in some cases of CMV retinitis, is associated with nephrotoxicity manifesting as proteinuria or elevated creatinine. (Choice D) Lamivudine is a nucleoside reverse transcriptase inhibitor (NRTI). Side effects are unusual with lamivudine, unlike other NRTIs. Lamivudine is occasionally associated with peripheral neuropathy and lactic acidosis. (Choice E) Sofosbuvir inhibits nonstructural protein 5B (NS5B), an RNA-dependent RNA polymerase needed for hepatitis C virus replication. Adverse effects associated with its use include fatigue and nausea. (Choice F) Valganciclovir is a ganciclovir prodrug and is unlikely to be useful against ganciclovir-resistant CMV. Ganciclovir is associated with severe neutropenia, which can be exacerbated by other bone-marrow suppressing agents (eg, zidovudine, trimethoprim-sulfamethoxazole). Educational objective: Foscarnet is an analog of pyrophosphate that can chelate calcium and promote nephrotoxic renal magnesium wasting. These toxicities can result in hypocalcemia and hypomagnesemia, which can cause seizures.

A 4-year-old boy is brought to the office with 3 days of fever, irritability, and oral lesions. His parents are concerned due to his refusal to eat or drink and lack of urination today. Physical examination demonstrates painful ulcers on the tongue and gingiva, swollen gums, and cervical lymphadenopathy. The remainder of his examination is unremarkable. Microscopic examination of an oral ulcer base scraping is shown in the image below. ID=1499 Multinucleated giant cell Which of the following is most likely the cause of this patient's condition? Abortive infection Latent infection Primary infection Slow virus infection Virus reactivation

The above image depicts a Tzanck smear, in which epithelial cells are scraped from an ulcer base and stained. The presence of multinucleated giant cells is suggestive of herpes simplex virus (HSV) infection, which is consistent with this child's symptoms. Tzanck smears are insensitive and do not differentiate between HSV and varicella-zoster virus infection. Therefore, Tzanck smears largely have been replaced by polymerase chain reaction testing. Primary infection with HSV-1 results in herpetic gingivostomatitis. Peak age for primary infection is age 6 months to 5 years. Prodromal symptoms (eg, fever, malaise, chills) begin approximately 1 week after contact with an infected person (who is generally asymptomatic). Painful vesicles appear, extensively covering the lips and gingiva, and may include the palate, tongue, or oropharynx. The vesicles organize into extensive ulcers and resolve within 1-2 weeks. The pain of gingivostomatitis often results in dehydration, the most common reason for hospitalization. (Choice A) An abortive viral infection refers to an infection in which the virus enters the cell but does not successfully produce new infective virus. Therefore, an abortive infection is not capable of causing disease. (Choice B) HSV is capable of latency by integrating into the trigeminal (or sacral, for HSV-2) ganglia. However, during latency no virus is produced and therefore no clinical disease occurs. (Choice D) A slow virus infection is characterized by a prolonged incubation period that lasts months to years. During this time, the virus gradually multiplies before finally causing clinically apparent disease. Examples of slow viral infections include HIV, hepatitis C, and subacute sclerosing panencephalitis following measles infection. (Choice E) Reactivation of HSV from the trigeminal ganglia causes herpes labialis. In contrast to gingivostomatitis, recurrent labialis disease is usually mild, unilateral, and limited to the lips due to host immune mechanisms (eg, presence of neutralizing antibody & HSV-specific cytotoxic T cells). Educational objective: Herpetic gingivostomatitis is a severe vesicular or ulcerative disease following primary infection with herpes simplex virus type 1. The diagnosis is supported by the presence of multinucleated giant cells in a Tzanck smear. Involvement of the gingiva, tongue, palate, and pharynx along with systemic symptoms (eg, fever, malaise) is common. In contrast, herpes reactivation in the trigeminal ganglia generally results in mild perioral vesicles.

A 38-year-old woman comes to the urgent care clinic with a 2-day history of sore throat. It is associated with fever and cervical lymphadenopathy, but not rhinorrhea, sneezing, or coughing. On examination, erythema and edema involving the posterior pharynx and a purulent tonsillar exudate are present. Rapid bedside streptococcal antigen testing is positive, and the patient is started on oral penicillin V. She returns the following day with a pruritic skin rash that came on suddenly after starting the antibiotic. The skin lesions observed on physical examination are shown in the image below. ID=1114; intensely pruritic, raised, erythematous plaques What is the most likely finding on biopsy of these lesions? Acanthyolysis Acanthosis Dermal edema Dyskeratosis Epidermal spongiosis

Urticaria ("hives") is a common, transient hypersensitivity disorder characterized by intensely pruritic, raised, erythematous plaques (wheals) that arise suddenly and resolve over several hours. They are most often caused by IgE-mediated degranulation of mast cells. They can also be due to non-IgE-mediated degranulation (eg, opiates, radiocontrast agents, physical urticaria) or mast cell-independent mechanisms (eg, aspirin, hereditary angioneurotic edema). Urticaria is due to increased permeability of the microvasculature, leading to edema of the superficial dermis. Involvement of the deep dermis and subcutaneous tissue is termed angioedema. Biopsy is rarely required for diagnosis. Findings on histopathology would typically include a mixed inflammatory infiltrate (mononuclear cells, neutrophils, and eosinophils) surrounding superficial dermal venules. Lymphatic channels may be engorged with transudative fluid. The overlying epidermis typically appears normal. (Choice A) Acantholysis is the loss of cohesion between keratinocytes in the epidermis. It is a characteristic finding in the pemphigus family of disorders. (Choice B) Acanthosis is a diffuse increase in thickness of the stratum spinosum (prickle cell layer) between the granular cell layer and the basal layer of the epidermis. Common conditions displaying acanthosis include psoriasis, seborrheic dermatitis, and acanthosis nigricans. (Choice D) Dyskeratosis describes the abnormal, premature keratinization of individual keratinocytes. Dyskeratotic cells are strongly eosinophilic and may have a small, basophilic nuclear remnant. Dyskeratosis can be found in diseases such as squamous cell carcinoma. (Choice E) Spongiosis is intercellular epidermal edema that histologically appears as an increase in the width of spaces between cells. Spongiosis is often associated with spongiotic dermatitis, which encompasses a number of conditions (eg, eczematous dermatitis). Educational objective: Urticaria is a transient hypersensitivity disorder characterized by pruritic erythematous plaques that arise suddenly and resolve over hours. They are most often caused by IgE-mediated degranulation of mast cells, leading to increased permeability of the microvasculature with edema of the superficial dermis.

A 36-year-old man comes to the office due to a 2-month history of a pruritic rash over the elbows and knees. The patient has been using skin emollients, but the lesions have not improved. He also has a prolonged history of episodic abdominal discomfort, flatulence, and voluminous greasy stools. Cardiopulmonary examination is normal. The abdomen is soft and nontender. Skin examination shows a papulovesicular skin rash in groups with erosion and excoriations. Which of the following is most likely to be seen in this patient? Crypt abscesses in colonic mucosa IgG-mediated skin disruption Increased intestinal intraepithelial lymphocytes Increased urinary prophyrines Insulin resistance

ID=1111 Dermatitis herpetiformis (DH) is characterized by erythematous pruritic papules, vesicles, and bullae that appear symmetrically on the extensor surfaces (eg, elbows, knees), upper back, and buttocks. Herpetiformis refers to the resemblance of the clustered vesicles to those of herpes simplex. DH is characterized histologically by microabscesses containing fibrin and neutrophils at the dermal papillae tips. The overlying basal cells become vacuolated, and coalescing blisters form at the tips of the involved papillae. Immunofluorescent imaging shows deposition of IgA at the dermoepidermal junction. The pathogenesis of DH is associated with that of celiac disease. Following absorption of dietary gluten, gliadin is deaminated by tissue transglutaminase in a process involving formation of covalent crosslinks between the two. Because of this, the subsequent immune response against gliadin also targets tissue transglutaminase, leading to the production of IgA and IgG tissue transglutaminase autoantibodies. In the skin, these antibodies cross-react with epidermal transglutaminase, and the resulting inflammation leads to the characteristic manifestations of DH. The majority of patients with DH exhibit the small-bowel histologic findings of celiac disease, which include: Small bowel intraepithelial lymphocytosis Crypt hyperplasia (elongation) Progressive villous atrophy (height shrinkage) (Choice A) Ulcerative colitis is characterized by mucosal inflammation, crypt abscesses (neutrophils in glandular lumen), and ulcerations. Common dermal associations include erythema nodosum, pyoderma gangrenosum, and psoriasis. The papulovesicular rash seen in this patient is not consistent with any of these entities. (Choice B) IgG-mediated skin disruption occurs in pemphigus vulgaris and bullous pemphigoid. These conditions are characterized by large bullae and ulcerations that are not confined to the extensor surfaces. Pemphigus vulgaris may involve the pharynx and esophagus, but neither condition is associated with steatorrhea. (Choice D) Elevated urinary porphyrin levels (ie, uroporphyrin, heptacarboxyl porphyrin) are seen in porphyria cutanea tarda. This condition is characterized by skin fragility and blistering lesions in sun-exposed areas. Steatorrhea would be unexpected. (Choice E) Insulin resistance occurs in type 2 diabetes mellitus, which can be associated with acanthosis nigricans and necrobiosis lipoidica diabeticorum. Although celiac disease and DH are associated with other autoimmune disorders, including type 1 diabetes, there is no association with type 2 diabetes. Educational objective: Dermatitis herpetiformis is characterized by erythematous pruritic papules, vesicles, and bullae that appear symmetrically on extensor surfaces. It is strongly associated with celiac disease, a disorder characterized histologically by small intestinal intraepithelial lymphocytosis, crypt hyperplasia, and villous atrophy.

A 6mm punch biopsy of normal appearing epidermis reveals stellar cells that contain intracytoplasmic granules having the shape of a tennis racquet. These cells demonstrate some myeloid surface markers and can interact closely with T lymphocytes. The cells described above are best referred to as which of the following? Kupffer cells Langerhans cells Merkel cells Melanocytes Macrophages Epithelioid cells

Langerhans cells (not to be confused with Langhans giant cells, a type of multinucleated cell formed from macrophages) are the form of dendritic cell most commonly found in the skin and mucous membranes. Dendritic cells are professional antigen presenting cells that interact closely with T lymphocytes. Dendritic are the most effective form of antigen presenting cell because they constitutively express MHC Class II and the co-stimulatory B7 cell surface molecules. They also take up antigen by constant endocytosis and pinocytosis from their environment. Langerhans cells are derived from the myeloid cell line and express myeloid cell surface markers. They contain characteristic "racquet-shaped" intracytoplasmic granules (Birbeck granules) that are only visible on electron microscopy. (Choice A) Kupffer cells are macrophage-derived cells present in the liver that compose part of the reticuloendothelial system. Kupffer cells lie within the hepatic sinusoids and, like any other macrophage, serve a phagocytic role in the liver. (Choice C) Merkel cells are neuroendocrine cells of the basal layer of the epidermis, often found in association with nerve projections, that play a role in perception of touch. They are associated with a rare but highly malignant form of skin cancer known as Merkel cell carcinoma. (Choice D) Melanocytes are pigment producing cells of the epidermis that arise from the neural crest cells and migrate to the basal layer of the epidermis during embryogenesis. They possess dendritic processes that intercalate between the remaining cells of the epidermis and facilitate the delivery of pigment-laden melanosomes from the melanocyte to the keratinocytes. (Choices E and F) Macrophages can be found in the skin, possess myeloid cell surface markers, and when stimulated, are able to produce dendrite-like extensions of their cytoplasm. Macrophages can also differentiate into multinucleated (Langhans) giant cells and epithelioid cells in the setting of granulomatous inflammation. However, monocytes and their derivatives do not produce Birbeck granules. Educational Objective: Langerhans cells are dendritic cells found in the skin that act as professional antigen presenting cells. These cells are derived from the myeloid cell line and they possess characteristic racquet-shaped intracytoplasmic granules known as Birbeck granules.

A 56-year-old woman comes to the office due to a pigmented lesion on her back. She first noted the lesion a year ago and says it has enlarged. Past medical history is unremarkable. The patient has worked as a forest service ranger for the last 30 years and has spent much of her time outdoors. On examination, there is a large irregular pigmented lesion on the upper back, as shown in the exhibit, as well as scattered patches of erythema and telangiectasias. Excisional biopsy of the lesion shows malignant melanoma. Which of the following is associated with the highest risk of metastasis for this patient's lesion? ID=1113 Active horizontal growth Active vertical growth Dense lymphocyte infilitration Multinucleated giant melanocytes Prominent cellular atypia

Malignant melanoma most often arises in the skin, although it may occur wherever there are pigmented cells, such as the eye and the mucosa of the gastrointestinal tract. Sunlight exposure appears to be an important factor in the pathogenesis of dermal melanoma, as are hereditary factors and a history of preexistent dysplastic nevi. Light-skinned individuals are at greatest risk. On gross inspection, melanomas are characterized by asymmetric shape, irregular or jagged border, variability of color (brown, black, red, blue, or unpigmented), diameter >0.5 cm, and evolution in size and appearance over time (ABCDE criteria). Histologically, melanoma cells congregate in poorly formed nests and are large with irregular nuclei, clumped chromatin, and prominent nucleoli. In early development, many melanomas exhibit a radial growth phase. This superficial, horizontal growth within the epidermis and superficial dermis carries a very low risk of metastatic spread (Choice A). In contrast, melanomas in the vertical growth phase contain melanoma cells that travel downward into the deeper dermal layers, increasing metastatic risk. Vertical growth often corresponds to a palpable nodularity of the lesion. The Breslow depth (distance from the epidermal granular cell layer to the deepest visible melanoma cells) is the most important prognostic indicator in malignant melanoma. (Choice C) A dense lymphocytic infiltration of the tumor suggests a robust immune response and is thought to improve prognosis. (Choice D) Multinucleated giant melanocytes are a characteristic finding in lentigo maligna melanoma. Lentigo maligna are common pigmented lesions in elderly patients and are considered to be a type of melanoma in-situ. When melanoma develops in lentigo maligna, the prognosis is similar to other melanomas of the same stage. (Choice E) Prominent cellular atypia is a classic finding of malignant melanoma, but it is not considered the most important risk factor for metastatic disease. Educational objective: Melanoma often has an early horizontal growth phase with low metastatic potential followed by a nodular, vertical growth phase with a significantly increased risk of metastasis. Depth of invasion (Breslow thickness) is the most important prognostic indicator in malignant melanoma.

A 28-year-old, previously healthy woman comes to the office for evaluation of a raised, pigmented skin lesion on her leg. The patient has had no itching or pain and cannot remember how long she has had the lesion. She has used tanning machines a few times but denies excessive sun exposure. The patient is particularly concerned about the risk for malignancy because her father was recently diagnosed with skin cancer. The skin lesion is shown in the image below: ID=11502 Biopsy of the lesion reveals nests of round, uniform cells at the basal portion of the epidermis that extend into the underlying dermis. The cells contain inconspicuous nucleoli and show no mitotic activity. Which of the following is the most likely diagnosis? Atypical nevus Compound nevus Junctional nevus Malignant melanoma Pigmented basal cell carcinoma Seborrheic keratosis

Melanocytic nevi are benign neoplasms composed of round, uniform melanocytes (nevus cells) that are mitotically quiescent. They are typically <6 mm in diameter and have a regular outline with a symmetric, sharply demarcated border and a homogeneous surface. Melanocytic nevi are progressive lesions that typically mature through the following phases: Junctional nevi are characterized by aggregates of nevus cells limited to the dermoepidermal junction (Choice C). They typically appear as flat, black to brown macules with darker color in the center than in the periphery and preserved skin markings. Compound nevi form as the aggregates of nevus cells extend into the dermis. They are raised papules with uniform brown to tan pigmentation. This patient's lesion is a compound nevus because it has both dermal and epidermal involvement. Intradermal nevi are older lesions in which the epidermal nests of nevus cells have been lost. The remaining dermal nevus cells lose tyrosinase activity and produce little to no pigment. Intradermal nevi are often skin-colored, dome-shaped, and sometimes pedunculated. (Choices A and D) Malignant melanoma is characterized by irregular and asymmetric borders with variable coloration, diameter ≥6 mm, and evolution over time. Melanomas typically expand superficially at first but may extend vertically into the dermis and subcutaneous tissues, portending a poor prognosis. Compared to nevus cells, melanoma cells are larger, contain irregular nuclei with prominent nucleoli, and have greater mitotic activity. Atypical (dysplastic) nevi can resemble melanoma clinically but typically have less extreme features. The melanocytes commonly show cellular atypia and form nests in the epidermis that fuse with nests in adjacent rete ridges. (Choice E) Pigmented basal cell carcinoma can present as papules, nodules, or plaques with rolled borders on sun-exposed areas. They can have varying degrees of pigmentation due to functional melanocytes within the lesion. The tumor cells resemble basal epidermal cells but extend into the dermis and often form nests with a palisading arrangement (ie, parallel alignment) of peripheral cells. (Choice F) Seborrheic keratosis forms a hyperpigmented lesion with well-circumscribed borders, a dull surface, and a "stuck-on" appearance. On microscopic examination, the lesions are composed of small cells resembling basal cells, with variable pigmentation, hyperkeratosis, and keratin-containing cysts. Educational objective: Compound nevi are benign proliferations of melanocytes that involve both the dermis and the epidermis. The lesions appear as slightly raised papules with uniform pigmentation and symmetric sharp borders.

A 34-year-old man comes to the physician due to fever, myalgias, malaise, and progressive fatigue over a 2-week period. He has not experienced any sore throat. He recently received a blood transfusion while hospitalized for a bleeding duodenal ulcer. Physical examination shows mild splenomegaly. There is no lymphadenopathy and no jaundice. Lymphocytosis is identified in the peripheral blood with 30% atypical lymphocytes. The patient's serum fails to agglutinate horse erythrocytes. The agglutination test is repeated a week later and, again, no agglutination is noted. HIV testing is negative. Which of the following is the most likely cause of this patient's condition? Coxsackievirus A Cytomegalovirus Epstein-Barr virus Hepatitis C virus JC virus Parvovirus B19

Most cases of infectious mononucleosis (IM), caused by the Epstein-Barr virus (EBV), are associated with serum heterophile antibodies that agglutinate with erythrocytes from unrelated species (eg, horse erythrocytes in the Monospot test, sheep erythrocytes in the classic Paul-Bunnell test). The agglutination test may be negative at the beginning of the infection and can be repeated later for confirmation. Because the Monospot (horse erythrocyte agglutination) test was negative on 2 separate occasions in this patient with a mononucleosis-like syndrome (fever, fatigue, splenomegaly, atypical lymphocytosis), his symptoms are unlikely to be due to EBV; the lack of sore throat and lymphadenopathy is also atypical for classic EBV-associated IM (Choice C). In immunocompetent patients with a heterophile antibody-negative mononucleosis-like syndrome, the most likely diagnosis is cytomegalovirus (CMV) infection. Pharyngitis and lymphadenopathy are seen less commonly than with EBV. CMV can be acquired during the transfusion of leukocyte-laden blood products, as the virus infects leukocytes of granulocyte-macrophage lineage. Irradiation helps reduce this risk. Other causes of heterophile antibody-negative mononucleosis-like syndromes include human herpesvirus-6, HIV, and toxoplasmosis. (Choice A) Coxsackie A viruses cause aseptic meningitis and herpangina (fever, posterior pharyngeal gray vesicles/ulcers) in children. (Choice D) Acute hepatitis C (HCV) rarely causes symptoms. Transfusion-acquired HCV can cause chronic hepatitis, but highly sensitive blood donor screening tests have led to low transmission rates. Atypical lymphocytosis is less common with HCV than with EBV or CMV. (Choice E) JC virus causes progressive multifocal leukoencephalopathy (PML), a central nervous system viral infection seen in immunocompromised patients. Clinical manifestations include hemiparesis, visual field defects (eg, hemianopsia), and cognitive impairment. (Choice F) Parvovirus B19 causes erythema infectiosum, a childhood illness characterized by red, flushed cheeks ("slapped cheeks" appearance). The virus can also cause aplastic crises in patients with sickle cell anemia and hydrops fetalis in the fetus. Educational objective: The Monospot test is positive in most cases of Epstein-Barr virus-associated mononucleosis. In immunocompetent patients with a heterophile antibody-negative mononucleosis-like syndrome, the most likely diagnosis is cytomegalovirus infection.

A 67-year-old man comes to the office after he noticed several nodules in his left axilla. He has a history of malignant melanoma on his upper back, which was treated with wide surgical excision 4 years ago. The patient undergoes biopsy of an axillary lesion, and histopathology reveals melanoma recurrence. Subsequent whole-body positron emission scan shows diffuse metastatic disease involving the lungs, liver, and left axillary nodes. Checkpoint inhibitor therapy with pembrolizumab is initiated, which leads to significant reduction in the size of the axillary nodules and metastatic lesions. Which of the following best correlates with the treatment response in this patient? Increase intratumor macrophages Intense desmoplastic reaction Ischemic tumor necrosis CD8+ lymphocyte infiltration Peritumoral neutrophilic reaction

Neoplastic cells produce proteins not found in healthy cells. Pieces of these proteins (neoantigens) are displayed on MHC class I molecules on the cell surface and are subsequently recognized by cytotoxic T cells as "nonself" (leading to cellular apoptosis). However, neoplastic cells generally accumulate genetic mutations that allow them to thwart the cytotoxic T-cell response and evade detection/destruction. One common mechanism is the overexpression of programmed death-ligand 1 (PD-L1) on the cancer cell surface; this ligand binds to the PD-1 receptor on cytotoxic T cells and suppresses their ability to induce apoptosis (T-cell exhaustion). Treatment with monoclonal antibodies that block PD-1 such as pembrolizumab have shown promise in cancers that overexpress PD-L1 (eg, melanoma, renal cell carcinoma). Blockade of PD-1 restores the cytotoxic T-cell response (disinhibition), which promotes tumor cell apoptosis. Patients with advanced melanoma and other susceptible malignancies often have a drastic clinical response (eg, tumor regression, long-term remission) to PD-1 antagonists. (Choices A and E) Cancer-related inflammation also involves cells of the innate immune system such as macrophages and neutrophils. These cells can have either protumor (eg, release of matrix metalloproteases that facilitate invasion) and antitumor (eg, direct killing of cancer cells) activities depending on the local cytokine milieu. However, PD-1 inhibitors do not increase the antitumor activity of neutrophils or macrophages. (Choice B) Tumor cells often induce desmoplasia, which refers to excessive connective tissue/stroma growth around the tumor. A strong desmoplastic response can reduce the ability of chemotherapeutic agents to penetrate into the tumor, leading to chemoresistance. (Choice C) Angiogenesis inhibitors (eg, bevacizumab) are used to prevent neovascularization of growing tumors, leading to ischemia and tumor necrosis. PD-1 inhibitors cause cytotoxic T-cell activation, leading to increased apoptosis of tumor cells; they do not induce ischemic necrosis. Educational objective: Programmed-death receptor 1 (PD-1) is a checkpoint inhibitor that downregulates the cytotoxic T-cell response. Neoplastic cells often exploit this receptor via the overexpression of PD-1 ligand. PD-1 receptor inhibitors (eg, Pembrolizumab) restore the T-cell response, allowing cytotoxic T cells to invade the tumor and induce apoptosis of neoplastic cells.

A 65-year-old man comes to the office due to skin lesions on his back. He first noted the spots 5 years ago and says they are becoming more numerous. The lesions are occasionally pruritic but are otherwise asymptomatic; he is not bothered significantly by them and made today's appointment at his wife's insistence. Medical history is notable for hypertension, hyperlipidemia, and type 2 diabetes mellitus. On examination, there are numerous pigmented plaques, as shown in the image below. ID=1112 Which of the following is the most likely diagnosis? Acanthosis nigricans Actinic keratosis Lichen planus Malignant melanoma Psoriasis Seborrheic keratosis

Seborrheic keratosis (SK) (plural: keratoses) is a common, tan or brown, epidermal tumor occurring in middle-aged or elderly individuals. It has a variable appearance from nearly flat macules to raised, wart-like lesions, ranging in size from a few millimeters to centimeters. SK typically has a velvety or greasy surface and well-demarcated border, and is often described as having a "stuck-on" appearance. On microscopic examination, the lesions of SK are composed of small cells resembling basal cells, with variable pigmentation, hyperkeratosis (thickening of the stratum corneum), and keratin-containing cysts. The etiology of SK is not fully understood; however, it is frequently associated with activating mutations of the fibroblast growth factor receptor 3. SK itself is benign, but rapid onset of numerous SKs (Leser-Trélat sign) is often associated with an internal malignancy (eg, gastric adenocarcinoma), possibly due to overproduction of insulin-like growth factor 1 and other cytokines. (Choice A) Acanthosis nigricans is characterized by hyperpigmented, velvety plaques at flexural areas (eg, axilla, posterior neck). It is associated with diabetes mellitus, obesity, and visceral malignancies. (Choice B) Actinic keratosis presents with hyperkeratotic lesions in sun-exposed areas of the scalp, ears, face, and hands. These lesions are unpigmented and, despite the similar name, are unrelated to seborrheic keratosis. (Choice C) Lichen planus is characterized by the "5 Ps": polygonal, planar, pruritic, purplish plaques on the wrists, hands, trunk, and legs. Fine white lines (Wickham striae) may be present on the plaque surface. (Choice D) Melanoma characteristically presents as an asymmetric, pigmented lesion with an irregular border, variable coloration, and change in size and appearance over time. Multifocal development of monotonous, otherwise asymptomatic lesions is much more typical of SK. (Choice E) Psoriasis presents with scaly, nonpigmented plaques. Although it may occur on the trunk, it is more common on the extensor surfaces of the extremities. Educational objective: Seborrheic keratosis is a common epidermal tumor that presents as a tan or brown, round lesion with a well-demarcated border and "stuck-on" appearance. Microscopic examination shows small cells resembling basal cells, with pigmentation, hyperkeratosis, and keratin-containing cysts. Rapid onset of numerous lesions is often associated with internal malignancy (Leser-Trélat sign).

A 23-year-old man is hospitalized for fever, headache, and confusion. He does not use tobacco or illicit drugs and is monogamous with his wife. The patient's cerebrospinal fluid analysis reveals lymphocytic pleocytosis, and a PCR is positive for herpesvirus. The patient is treated with intravenous high-dose acyclovir. Fever and mental status gradually improve, but on the third day of hospitalization the patient's serum creatinine level increases to 3.4 mg/dL from a baseline of 0.9 mg/dL at admission. The observed finding could have been prevented by which of the following? Aggressive intravenous hydration Careful taking of allergy history Monitoring of blood-drug levels Pretreatment with allopurinol Pretreatment with prednisone

The acute elevation of this patient's serum creatinine on the third day of hospitalization is likely due to acyclovir nephrotoxicity, which occurs in 5%-10% of patients who receive the drug intravenously. Acyclovir is excreted principally in the urine via glomerular filtration and tubular secretion. When the acyclovir concentration in the collecting duct exceeds its solubility, crystallization, crystalluria, and renal tubular damage can occur. In most cases, this toxic complication is transient and can be prevented (as well as treated) with adequate hydration and a reduction in the rate of drug infusion. (Choice B) Acute interstitial nephritis (AIN) is an allergic inflammation of the kidney that is most commonly triggered by medications and is often associated with fever, rash, and eosinophilia. However, acyclovir is not a common cause of AIN; common causes include nonsteroidal anti-inflammatory drugs, penicillins, cephalosporins, diuretics, and allopurinol. (Choice C) Monitoring of acyclovir blood levels is not always effective in preventing nephrotoxicity. During states of low intravascular volume (eg, dehydration), nephrotoxicity can develop secondary to increased urinary concentration even when acyclovir blood levels are not within the established toxic range. (Choice D) Allopurinol blocks xanthine oxidase, which reduces the formation of uric acid. Pretreatment with allopurinol is often used in patients with lymphoma and leukemia to prevent urate crystal nephropathy due to tumor lysis syndrome. Acyclovir nephrotoxicity is mediated by the formation of acyclovir crystals, not urate crystals, so allopurinol pretreatment would be ineffective. (Choice E) Pretreatment with prednisone can help prevent radiocontrast-induced allergic reactions. Prednisone does not prevent the formation of acyclovir crystals. Educational objective: Intravenous acyclovir can cause crystalline nephropathy if adequate hydration is not also provided.

A 53-year-old man comes to the emergency department due to progressively worsening shortness of breath, nonproductive cough, and low-grade fevers over the past 2 weeks. He has not had a runny nose or sore throat and does not recall any sick contacts. He received a lung transplant for idiopathic pulmonary fibrosis 4 months ago. His medications include immunosuppressants and trimethoprim-sulfamethoxazole. Temperature is 37.8 C (100 F). Chest x-ray reveals diffuse interstitial infiltrates bilaterally. A decrease in pulmonary function is noted on testing. A lung biopsy specimen is shown below. ID=1576 "Owl's eyes" Which of the following best characterizes the organism most likely responsible for this patient's current condition? Enveloped double-stranded DNA virus Enveloped single-stranded RNA virus Filamentous gram-positive rods Nonenveloped double-stranded DNA virus Nonenveloped single-stranded RNA virus Trophic and cystic fungal forms

Transplant patients are at risk for a variety of unusual infections due to their immunocompromised state. Cytomegalovirus (CMV) is particularly common in patients with lung transplants (typically occurring within the first few months after transplant). Most but not all transplant centers practice universal prophylaxis for lung transplant recipients (eg, valganciclovir). CMV is an enveloped double-stranded DNA virus belonging to the Herpesviridae family. Major risk groups for infections include transplant patients, patients with HIV, and fetuses (congenital infections). CMV pneumonitis is the most common form of tissue-invasive CMV following lung transplantation; other organ-specific disease manifestions (eg, esophagitis, colitis, and retinitis) occur more frequently in patients with HIV. Biopsy findings consistent with CMV include enlarged cells with intranuclear and intracytoplasmic inclusions (viral particles); there is often a surrounding halo (owl's eye). (Choices B, D, and E) Influenza viruses are enveloped single-stranded RNA viruses. Rhinovirus (a cause of the common cold) is a nonenveloped single-stranded RNA virus. Adenovirus is a nonenveloped double-stranded DNA virus. Although these viruses can cause respiratory illness, this patient's clinical features (2 weeks of progressive symptoms, impaired pulmonary function, no upper respiratory infection symptoms) and histologic findings make post-transplant CMV pneumonitis a more likely diagnosis, particularly given the lack of CMV prophylaxis. (Choice C) Organs affected by Nocardia and Actinomyces show filamentous branching beaded gram-positive rods in Gram stain preparations. Nocardia is aerobic and positive for modified acid-fast stain; Actinomyces is anaerobic and negative for modified acid-fast stain. (Choice F) Histopathologic and cytologic findings in Pneumocystis jirovecii pneumonia (PCP) include eosinophilic foamy alveolar material and cystic and trophic forms that can be stained with Giemsa or silver stain (producing a cup-in-saucer appearance). In addition, PCP is more common in patients with advanced HIV than in transplant patients and would be unlikely in this patient on PCP prophylaxis with trimethoprim-sulfamethoxazole. Educational objective: In a transplant patient, pneumonia with intranuclear and cytoplasmic inclusion bodies histologically points to opportunistic infection with cytomegalovirus, an enveloped virus that contains a double-stranded DNA genome.

A 64-year-old man comes to the office with a 4-day history of severe left-sided chest discomfort. The pain is constant and has a burning quality. Over the last day, the patient has developed a rash in the area of discomfort. He has not had similar symptoms before. Past medical history is unremarkable. The patient is afebrile and vital signs are normal. Physical examination shows a vesicular rash at the 5th left intercostal space. Which of the following pathologic findings is most likely to be found in the affected area? Acantholysis and intercellular IgG deposits Eosinophilic cytoplasmic inclusions Koilocytosis of the superficial epidermal layers Microabscesses at the tips of the dermal papillae Multinucleated giant cells with intranuclear inclusions

Varicella zoster virus (VZV) is an enveloped, double-stranded DNA virus transmitted via respiratory droplets or direct contact. Initial infection typically occurs in childhood and causes varicella (chicken pox), which is characterized by fever and a self-limited, diffuse vesicular rash. VZV then travels via sensory fibers to the dorsal root ganglia (or trigeminal ganglia), where it remains dormant for years. Weakening of cellular immunity leads to reactivation of the virus, which manifests as herpes zoster (shingles). Shingles is characterized by unilateral burning pain and a papular or vesicular rash in a dermatomal distribution. The lesions coalesce, rupture, crust over, and heal within a few weeks although the discomfort may linger for several weeks. Light microscopy of a sample from a vesicle base reveals intranuclear inclusions in keratinocytes and multinucleated giant cells (positive Tzanck smear). Skin biopsy would show acantholysis (loss of intercellular connections between keratinocytes) and intraepidermal vesicles. (Choice A) Acantholysis forming suprabasal blisters is characteristic of pemphigus vulgaris. Immunofluorescence reveals deposition of IgG-containing deposits in a reticular pattern around keratinocytes (targeting desmosome protein desmoglein 3). (Choice B) Infection by molluscum contagiosum, a poxvirus, causes eosinophilic cytoplasmic inclusions (molluscum bodies) in infected cells and dome-shaped, umbilicated papules on examination. (Choice C) Human papillomavirus infection of the skin presents with warts (verruca vulgaris) or squamous cell carcinoma of the penis or vulva, depending on the serotype. Light microscopy reveals cytoplasmic vacuoles in keratinocytes (koilocytosis) and epidermal hyperplasia. (Choice D) Dermatitis herpetiformis presents with pruritic, grouped vesicles on the extensor surfaces. Light microscopy reveals accumulations of neutrophils on the tips of dermal papillae (microabscesses). Educational objective: Herpes zoster (shingles) develops due to reactivation of varicella zoster virus in the dorsal root ganglia (sensory neurons). It presents with a painful vesicular rash in a dermatomal distribution. Intranuclear inclusions in keratinocytes and multinucleated giant cells are seen on light microscopy.

A 64-year-old man comes to the office due to blisters on his trunk and groin. Itching has occurred for the past several weeks, and blisters developed a week ago. He has a history of hypertension and osteoarthritis. The patient received the zoster vaccine at age 60. Examination shows numerous 0.5- to 3-cm bullous skin lesions. The findings from a skin biopsy are shown in the image below: ID=1105 Which of the following is the most likely diagnosis? Bullous pemphigoid Contact dermatitis Dermatitis herpetiformis Lichen planus Pemphigus vulgaris

Bullous pemphigoid (BP) is an autoimmune blistering disease characterized by serous fluid-filled, tense bullae on normal or erythematous skin. The most common sites of involvement include the inner thighs and flexor aspects of the forearms, axillae, groin, and lower abdomen. Elderly individuals are most commonly affected. BP is caused by antibodies against hemidesmosomes along the basement membrane of the dermal-epidermal junction. These autoantibodies result in a destructive inflammatory cascade that causes the entire epidermis to separate from the dermis, forming subepidermal, nonacantholytic blisters. Immunofluorescence shows IgG and/or C3 deposits in a linear pattern along the basement membrane. (Choice B) Acute allergic contact dermatitis is characterized by localized erythema, vesicles, edema, and severe pruritus. Biopsy typically shows spongiosis (abnormal accumulation of edema fluid in the intercellular spaces between keratinocytes) and lymphocytic perivascular infiltrates but no separation of the epidermis from the dermis. (Choice C) Dermatitis herpetiformis presents with small, pruritic papules and vesicles over extensor surfaces and is characterized histologically by microabscesses containing fibrin and neutrophils at the dermal papillae tips. The overlying basal cells become vacuolated, and blisters form at the tips of the involved papillae. (Choice D) Lichen planus causes pruritic, purple papules and plaques involving the skin and mucous membranes (not blisters). Microscopic analysis shows a lymphocytic infiltrate at the dermal-epidermal junction with destruction of basal epidermal cells. (Choice E) Pemphigus vulgaris is due to autoantibodies directed against desmosomal proteins. Biopsy of an active pemphigus lesion shows intraepithelial cleavage with detached keratinocytes (acantholysis), retained keratinocytes along the basement membrane, and an eosinophilic inflammatory infiltrate. Educational objective: Bullous pemphigoid is caused by autoantibodies against hemidesmosomes along the basement membrane of the dermal-epidermal junction. This causes the entire epidermis to separate from the dermis and form tense, subepidermal blisters.

A 45-year-old woman is brought to the emergency department after a generalized tonic-clonic seizure. She reports no prior history of seizures, but she has noticed right arm weakness over the last week. Her family history is significant for her mother's death from skin cancer. Physical examination shows a skin lesion measuring 13 mm on her back, as shown in the image below. ID=1960 This lesion most likely originated from which of the following embryologic derivatives? Endoderm Mesoderm Neural crest Neuroectoderm Surface ectoderm

Clinical features of melanoma (ABCDE) Asymmetry: when bisected, the 2 sides are not identical Border irregularities: uneven edges, pigment fading off Color variegation: variable mixtures of brown, tan, black & red Diameter: ≥6 mm Evolving: lesion changing in size, shape, or color; new lesion This patient's family history of skin cancer, the melanocytic lesion seen on skin examination, and her 1-week history of focal neurologic deficit suggest that her seizure results from malignant melanoma metastatic to the central nervous system. Melanoma commonly metastasizes to the brain, gastrointestinal tract, bone, liver, and lungs. It is a malignancy of melanocytes, which are of neural crest origin. (Choice A) The endoderm gives rise to all structures derived from the inner lining of the primitive gut tube. These include the thyroid follicular cells; epithelial surfaces of the trachea, bronchi, and lungs; liver and biliary tree; pancreas; and gastrointestinal and bladder epithelium. (Choice B) The mesoderm gives rise to the dermis as well as most bones, muscles, blood vessels, and visceral tissue. (Choice D) The neuroectoderm gives rise to the central nervous system, preganglionic autonomic neurons, retina, and posterior pituitary. (Choice E) The surface ectoderm gives rise to the epidermis and its appendages, mammary glands (modified sweat glands), lens of the eye, and adenohypophysis. Educational objective: The most common metastatic tumors to the brain are lung cancer, renal cancer, and melanoma. Melanoma is a malignancy of melanocytes, which are embryologically derived from neural crest cells.

A 64-year-old man who recently moved to the area comes to the office for a new patient evaluation. He has chronic low back pain, resulting from an injury 8 years ago, for which he uses several over-the-counter analgesics. Most recently, the patient has been taking naproxen daily. He has no other joint pain, fever, rash, urinary symptoms, or other medical conditions. Blood pressure is 135/70 mm Hg and pulse is 78/min. Examination shows trace lower extremity edema. Neurologic and musculoskeletal examinations reveal no abnormalities. Laboratory results show blood counts within normal limits, blood urea nitrogen of 12 mg/dL, and serum creatinine of 2.0 mg/dL. Urinalysis reveals 1+ protein and 3-4 white blood cells/hpf. Renal ultrasound demonstrates bilateral shrunken and irregular kidneys with a few papillary calcifications. Which of the following is the most likely cause of this patient's renal dysfunction? Chronic interstitial nephritis Chronic pyelonephritis Crystal nephropathy Focal segmental glomerular sclerosis Ischemic tubular necrosis Renal artery stenosis

ID=1049 Over-the-counter analgesics such as nonsteroidal anti-inflammatory drugs (NSAIDs) can cause renal failure (analgesic nephropathy) if taken in large amounts over extended periods. Affected patients typically have a modest elevation in serum creatinine, mild proteinuria, and evidence of tubular dysfunction (polyuria, nocturia). Microscopic hematuria and sterile pyuria (white cells without bacteria) may also be seen on urinalysis. NSAIDs concentrate in the renal medulla along the medullary osmotic gradient, with higher levels in the papillae. These drugs uncouple oxidative phosphorylation and increase oxidative stress, resulting in damage to tubular and vascular endothelial cells. Prolonged use results in chronic interstitial nephritis visualized as patchy interstitial inflammation with subsequent tubular atrophy and fibrosis, papillary necrosis, and scarring. Grossly, the kidneys appear shrunken with irregular contours and distortion of the caliceal architecture. NSAIDs also decrease prostaglandin synthesis, causing renal vasoconstriction which further increases the risk of ischemic papillary necrosis. (Choice B) Chronic pyelonephritis can also cause chronic interstitial nephritis with papillary necrosis. However, urinalysis would be expected to demonstrate evidence of infection (eg, bacteria, nitrites, marked pyuria), and the patient would likely have flank pain and fever. (Choice C) Acute crystal nephropathy presents with renal failure, and patients often have nausea, vomiting, or flank pain. Chronic crystalline nephropathy is most commonly seen in patients with gout or hyperuricemia secondary to urate deposition in the medullary interstitium. (Choice D) Focal segmental glomerular sclerosis (FSGS) presents with nephrotic-range proteinuria. Drug-related secondary FSGS has been reported with the use of anabolic steroids or heroin but not NSAIDs. (Choice E) Ischemic tubular necrosis usually occurs acutely after a prolonged period of hypotension (eg, sepsis, major surgery); muddy brown casts would be expected on urinalysis. (Choice F) Symptoms of renal artery stenosis include resistant hypertension, recurrent flash pulmonary edema, and chronic kidney disease. The absence of hypertension and history of NSAID use are more consistent with chronic interstitial nephritis . Educational objective: Analgesic nephropathy is a form of chronic kidney disease caused by prolonged, heavy intake of nonsteroidal anti-inflammatory drugs and/or acetaminophen. Pathologic characteristics include chronic interstitial nephritis and papillary necrosis.

A 28-year-old man comes to the office with bilateral skin lesions over the posterior surface of the elbows. The lesions are itchy at times; topical moisturizers have reduced the itching, but the lesions have remained. The patient has no chronic medical problems and takes no medications. Biopsy of the lesions demonstrates hyperkeratosis, parakeratosis, dilated capillaries in the dermal papillae, and foci of neutrophils in the superficial epidermis. Which of the following is the most likely diagnosis? Atopic dermatitis Dermatitis herpetiformis Erythema nodosum Psoriasis Urticaria

ID=1117 Psoriasis is a common chronic inflammatory skin disorder that presents with scaly, erythematous plaques, typically on the extensor surfaces of the elbows and knees. In predisposed individuals, disruption of the epithelial barrier leads to activation of antigen-presenting dendritic cells and subsequently to a self-reinforcing inflammatory cascade characterized by recruitment and activation of T helper cells and proliferation of keratinocytes. Histopathologically, psoriasis is characterized by: Hyperkeratosis (thickening of the stratum corneum) and confluent parakeratosis (retention of nuclei in the stratum corneum) produces the characteristic silvery scale on gross examination Epidermal hyperplasia (acanthosis) with elongated rete ridges correspond to the typical erythematous plaques Neutrophilic foci in the stratum corneum and epidermis, which may coalesce to form microabscesses (Munro microabscesses) In addition, the epidermal cell layer superficial to the dermal papillae may be thinned and contain dilated blood vessels, which leads to pinpoint bleeding when the scale is removed from the plaque (Auspitz sign). (Choice A) Chronic atopic dermatitis can produce plaques characterized by hyperkeratosis and acanthosis, often with a lymphocytic or eosinophilic infiltrate. However, adults most commonly develop lesions on flexural surfaces (eg, anterior elbows), and most patients have a history of other atopic disorders (eg, asthma, allergic rhinitis) beginning in childhood. (Choice B) Dermatitis herpetiformis presents with erythematous, pruritic papules, vesicles, and bullae symmetrically on the elbows, knees, upper back, and buttocks. It is strongly associated with celiac disease, and histopathology shows microabscesses at the dermal papillary tips, which coalesce to form subepidermal blisters. (Choice C) Erythema nodosum is characterized by painful, red nodules on the shins and often represents an inflammatory response to medications (eg, oral contraceptives), infections, inflammatory bowel disease, or sarcoidosis. Histopathology is variable but typically shows a septal panniculitis with a mixed cellular infiltrate. (Choice E) Urticaria ("hives") is a transient dermatologic disorder that is caused by antigen-induced degranulation of focal mast cells through IgE antibody sensitization. This degranulation causes microvasculature hyperpermeability and the formation of wheals that typically surface and resolve within hours. Educational objective: Psoriasis is characterized by hyperkeratosis and confluent parakeratosis of the stratum corneum, and epidermal hyperplasia (acanthosis) with elongated rete ridges. Neutrophilic foci in the stratum corneum and epidermis may coalesce to form microabscesses (Munro microabscesses).

A 17-year-old boy comes to the office for evaluation of a rash. For the past 2 years, the patient has had recurrent skin bumps that are frequently red and painful. He has no other medical conditions and takes no medications. Vital signs are within normal limits. Skin examination findings are shown below. ID=15374 This patient's skin condition primarily affects which of the following structures? Dermal papillae Eccrine sweat galnds Epidermal-dermal junction Melanocytes Pilosebaceous follicles Subcutaneous adipose tissue

ID=1641 missing This patient has typical features of acne vulgaris. Acne usually appears in adolescence and can present with a variety of lesions, including comedones, papules, pustules, or inflammatory nodules, often with lesions in different stages concurrently. Nodular (sometimes called nodulocystic) acne, as in this patient, is often painful, difficult to treat, and can lead to the formation of sinus tracts and residual scarring. Acne vulgaris is characterized by inflammation of pilosebaceous follicles. Contributing factors include hyperkeratinization and obstruction of follicles; sebaceous gland enlargement with increased sebum secretion; colonization of the gland by Propionibacterium (Cutibacterium) acnes, which metabolizes lipids in sebum and produces proinflammatory factors; and follicular and perifollicular inflammation with follicular rupture. (Choice A) Dermatitis herpetiformis most commonly occurs in patients with celiac disease and is characterized by microabscesses containing fibrin and neutrophils at the dermal papillae tips. Clinical manifestations include pruritic papules, vesicles, and bullae that appear symmetrically on the extensor surfaces (eg, elbows, knees), upper back, and buttocks. (Choice B) Eccrine sweat glands produce watery sweat in response to exertion, high ambient temperature, and emotional stress. The secretions from eccrine glands have little lipid content and do not support heavy colonization by P acnes; therefore, these glands do not have a prominent role in the pathogenesis of acne. (Choice C) Bullous pemphigoid is characterized by an autoimmune process that causes the epidermis to separate from the dermis, forming subepidermal blisters. It presents with tense, fluid-filled bullae on the inner thighs, forearms, axillae, groin, and abdomen. (Choice D) Melanocyte disorders typically present with hyperpigmentation (eg, melasma) or hypopigmentation (eg, vitiligo). (Choice F) Erythema nodosum—characterized by painful, subcutaneous nodules, typically in the lower legs—is a form of panniculitis (inflammation of the subcutaneous adipose tissue) caused by a delayed hypersensitivity reaction to antigens associated with various conditions (eg, herpes simplex). Educational objective: Acne vulgaris is an inflammatory disorder of pilosebaceous follicles. Contributing factors include hyperkeratinization and obstruction of follicles; sebaceous gland enlargement with increased secretion of sebum; colonization and proliferation in the gland by Propionibacterium (Cutibacterium) acnes; and follicular and perifollicular inflammation.

A 56-year-old man is brought to the emergency department with new-onset tonic-clonic seizures. His past medical history is unremarkable. The patient recently has had right-sided headaches and has taken acetaminophen with some relief. MRI of the brain reveals several round lesions in the right temporal lobe. After initial evaluation, a stereotactic biopsy of one of the lesions is performed. The biopsy results show neoplastic tissue containing a mutation in the gene that encodes BRAF, a protein kinase. The point mutation results in a valine → glutamic acid substitution at position 600 of the protein. Which of the following is the most likely diagnosis? Glioblastoma Melanoma Prostate cancer Renal cell carcinoma Small cell lung cancer

ID=7707 The main environmental risk factor for cutaneous melanoma is increased exposure to ultraviolet radiation (eg, solar or tanning bed sources), which can cause malignant transformation of melanocytes. The risk of melanoma is also increased by mutations in specific genetic factors, most commonly tumor suppressor genes and various oncogenes. BRAF is a protein kinase involved in activating the signaling pathways of melanocyte proliferation. BRAF mutation V600E (valine → glutamic acid), seen in 40%-60% of patients with melanoma, leads to greatly increased activation of the signaling pathways for melanocyte growth, survival, and metastasis. Vemurafenib, a potent inhibitor of mutated BRAF, has significant anti-tumor effects with improved survival and long-term outcomes in advanced stage V600E-positive melanoma patients. The presence of this BRAF mutation in this patient's tissue sample most likely indicates a diagnosis of metastatic melanoma. (Choice A) Glioblastoma is more likely to present as a single lesion; BRAF mutations are less common than with melanoma. (Choice C) Prostate cancer is associated with BRCA1 and BRCA2 mutations, not BRAF mutations. BRCA mutations also increase the risk of breast cancer. (Choices D and E) Renal cell carcinoma and small cell lung cancer appear to be multifactorial in origin, involving many different genes and environmental factors. However, BRAF has not been associated with either malignancy. Educational objective: BRAF is a protein kinase involved in activation of signaling pathways for melanocyte proliferation; the BRAF V600E mutation is seen in 40%-60% of patients with melanoma.

A 52-year-old man is evaluated for low-grade fever, malaise, abdominal pain, and diarrhea. The patient underwent a cadaveric renal transplant 6 months ago to treat renal failure from underlying polycystic kidney disease. He takes maintenance immunosuppressive therapy. Physical examination shows mild tenderness in the lower abdominal quadrants with no rebound tenderness. Serum immunosuppressant levels are within therapeutic range, and renal function studies are normal. Colonoscopy reveals mucosal erythema and ulcerations. Biopsy shows large cells with inclusion bodies, and a polymerase chain reaction-based test detects viral DNA. First-line therapy for the infection is initiated. This patient is at greatest risk of which of the following from the treatment? Hepatitis Myopathy Neutropenia Pancreatitis Thyroiditis

Patients who undergo organ transplantation and are on immunosuppressive therapy are at risk for cytomegalovirus (CMV) reactivation with end-organ disease (eg, colitis, retinitis, pneumonitis). This patient has classic manifestations of CMV colitis including fever, fatigue, abdominal pain, and diarrhea. Because serum testing for CMV DNA may be negative, patients with suspected CMV colitis usually undergo colonoscopy with biopsy. Typical findings include colonic mucosal erythema/ulcerations. Biopsy generally reveals large cells with intranuclear and intracytoplasmic inclusions ("owl's-eye" appearance). First-line therapy for CMV colitis is intravenous ganciclovir. This guanine nucleoside analogue interferes with viral DNA replication by competitively inhibiting the incorporation of guanosine triphosphate into CMV DNA strands by CMV DNA polymerase. However, this drug also impedes human DNA polymerase to a lesser degree and can interfere with host cell replication, particularly in areas of high turnover (eg, hematologic progenitor cells). Therefore, ganciclovir is often associated with prominent hematologic side effects such as neutropenia, anemia, and thrombocytopenia. (Choice A) The tuberculosis medications isoniazid and pyrazinamide can cause significant hepatotoxicity. (Choice B) Myopathy and rhabdomyolysis can be associated with the use of integrase inhibitors (eg, raltegravir, dolutegravir) in the treatment of HIV. (Choice D) Pancreatitis is an important side effect of the HIV medication didanosine, a nucleoside reverse transcriptase inhibitor. (Choice E) Amiodarone is an antiarrhythmic medication that can cause hypothyroidism, hyperthyroidism, or thyroiditis. Educational objective: First-line therapy for CMV colitis and retinitis is ganciclovir. This medication inhibits viral synthesis by blocking CMV DNA polymerase. However, ganciclovir also blocks host DNA polymerase to a lesser degree, which can lead to hematologic side effects such as neutropenia, anemia, and thrombocytopenia.

A 43-year-old man comes to the emergency department due to painless, progressive visual impairment. The patient was diagnosed with sexually acquired HIV infection 4 years ago. He was previously started on antiretroviral therapy but has been noncompliant with his prescribed medications. His most recent CD4 count was 37 cells/mm3. Physical examination shows no skin rash or corneal lesions. Funduscopy reveals the findings seen in the image below. ID=1646 Which of the following medications is the best initial therapy for this patient? Acyclovir Amphotericin B Clarithromycin Flucytosine Ganciclovir Penicillin G

Patients with untreated AIDS who have CD4 counts <50/mm3 are at high risk for cytomegalovirus (CMV) retinitis. Suspicion is often raised when a patient with advanced AIDS develops slowly progressive blurred vision, scotoma (blind spots), floaters, or photopsia (flashing lights). Although most cases are thought to arise due to hematogenous dissemination of CMV, serum testing for the virus is often (~50%) negative. Therefore, the diagnosis is typically established when funduscopy reveals yellow-white, fluffy retinal lesions near the retinal vessels associated with hemorrhage. Treatment is required to prevent progression and blindness. Ganciclovir, a guanine nucleoside analogue that is structurally similar to acyclovir but expresses greater activity against CMV DNA polymerase, is first-line therapy. Patients are also typically started on antiretroviral therapy to increase CD4 count. A major complication of CMV retinitis is retinal detachment due to the replacement of damaged retina with thin, atrophic scar tissue (which is prone to tear). (Choice A) Acyclovir is used to treat herpes simplex (HSV) and varicella zoster virus (VZV) infections. Although these infections occasionally cause acute retinal necrosis, most cases arise in older, immunocompetent individuals (not patients with advanced AIDS). In addition, HSV retinitis and VZV retinitis are usually associated with significant vitreal inflammation, spotty peripheral retinal lesions, and infrequent hemorrhage. (Choice B) Candida endophthalmitis is treated with amphotericin B. Funduscopy usually reveals focal, white, mound-like lesions on the retina. Most cases occur in the setting of an indwelling central venous catheter or total parenteral nutrition. (Choice C) Clarithromycin is used in conjunction with other antibiotics for the treatment of Mycobacterium avium complex infection, which usually causes fever, night sweats, abdominal pain, and diarrhea. (Choice D) Cryptococcus neoformans infections are treated with flucytosine, a fluorinated analog of cytosine, in combination with amphotericin B. Although cryptococcal endophthalmitis can occur in patients with advanced AIDS, it is much less common than CMV retinitis, and patients often have other manifestations of cryptococcal infection such as skin lesions or meningoencephalitis. (Choice F) Penicillin G is used to treat ocular (and neuro-) syphilis. Ocular syphilis often presents as uveitis but can cause retinitis, which usually appears as ground-glass lesions with significant vitreal inflammation. Most patients also have manifestations of syphilitic meningitis (eg, headache, vomiting). Educational objective: Cytomegalovirus retinitis is the most common cause of ocular disease in patients with untreated AIDS who have CD4 counts <50/mm3. Diagnosis is made by funduscopy, which typically reveals yellow-white, fluffy retinal lesions near the retinal vessels with associated hemorrhage. Treatment with ganciclovir is required to prevent blindness.

A 68-year-old woman comes to the office for evaluation of a skin rash. The patient says that red, painful papules erupted on her right chest 5 days ago and quickly turned into vesicles containing clear fluid. The lesions are now beginning to crust and are less painful. She has never had similar lesions and does not recall coming into contact with a person having a similar illness. Medical history is significant only for well-controlled hypertension. Physical examination shows a grouped vesicular rash involving the right T4 dermatome that is beginning to heal with crusting. The rash does not cross the midline. Lightly touching the skin elicits sharp pain. The remainder of the examination shows no abnormalities. This patient is at greatest risk of developing which of the following complications as a result of her current condition? Dissemination of rash to other body areas Painful swelling of the large joints Persistent pain in the affected region Recurrent episodes of gross hematuria Sensory loss in "glove and stocking" pattern

This patient has a vesicular rash consistent with herpes zoster (shingles), which is caused by reactivation of latent varicella-zoster virus (VZV) infection. Following the primary infection (chickenpox), VZV remains dormant in the dorsal root ganglia until emerging decades later. Decreased cell-mediated immunity (eg, older age, immunosuppressive medications, HIV infection) increases the risk of reactivation. Reactivation of VZV causes hemorrhagic inflammation of the dorsal root ganglion and peripheral nerve. The resulting hypersensitive nerve function presents with neuropathic pain in the affected region (herpetic neuralgia), typically described as stabbing, tingling, or burning. Herpetic neuralgia can begin even before the appearance of skin lesions and may persist for up to several months. A minority of patients experience long-term residual pain, termed postherpetic neuralgia; the risk is greater in older patients (age >70) and those with more severe shingles. (Choice A) Primary VZV infection is characterized by a diffuse vesicular rash (chickenpox), but reactivation is dermatomal. Generalized reactivation or spread to other body areas can occur in immune-compromised patients but is extremely rare in healthy individuals. (Choice B) Reactive arthritis presents with an asymmetric peripheral oligoarthritis, often with extraarticular symptoms (eg, uveitis, conjunctivitis, urethritis). It typically follows bacterial gastrointestinal (eg, Salmonella, Shigella, Campylobacter) or genitourinary (eg, Chlamydia) infection. (Choice D) IgA nephropathy produces recurrent glomerulonephritis and typically presents with episodic hematuria. Common triggers include upper respiratory illness, but shingles does not typically exacerbate the condition. (Choice E) Toxic or metabolic nerve injury (eg, diabetic peripheral neuropathy) is typically proportionate to the length of the sensory nerve fibers. Symptoms appear first in the longest nerves, such as those to the feet and hands, causing a "glove and stocking" distribution. However, because reactivation of VZV occurs only in specific sensory ganglia, symptoms have a dermatomal distribution. Educational objective: A unilateral vesicular rash localized on a single dermatome in an older patient is most likely herpes zoster. Postherpetic neuralgia is the most common neurologic complication of varicella zoster virus infection.

A 58-year-old man comes to the physician for evaluation of skin lesions on his forehead. He says that he first noticed the lesions when he was rubbing his forehead because of their roughness. His medical problems include hypertension that is well controlled with chlorthalidone. He has worked as a construction foreman for 32 years. Physical examination shows the findings seen in the image below. ID=935 erythematous papules with overlying whitish scale; sandpaper-like texture Which of the following is the most likely diagnosis? Actinic keratosis Atopic dermatitis Pityriasis rosea Psoriasis Seborrheic keratosis

This patient has actinic keratoses (AKs), which most commonly present as erythematous papules with overlying whitish scale. They are often "felt more than seen" and have a rough, sandpaper-like texture on palpation. The lesions are small and flat at first, but may enlarge and become elevated. Usually their size does not exceed 10 mm in diameter. Hyperkeratosis in such lesions may become prominent and form cutaneous horns. AKs develop in genetically predisposed individuals age 40-60 under the influence of excessive sun exposure. The most commonly affected areas are the face, ears, scalp, and the dorsa of the arms and hands, but any other chronically sun-exposed site (legs, back, upper chest) can be involved. AK is regarded as a premalignant condition, but less than 1% of AKs per year will evolve into invasive squamous cell carcinoma. (Choice B) Acute atopic dermatitis occurs most often in infants and children and manifests with highly pruritic erythematous papules and plaques. Light microscopy demonstrates spongiosis (edema of the epidermis). (Choice C) Pityriasis rosea often begins as a solitary pink or brown scaly plaque with central clearing on the trunk, neck, or extremities (herald patch). It is followed by development of an ovoid maculopapular rash with lesions classically oriented in an oblique direction along the skin tension lines on the back (Christmas tree pattern). (Choice D) Psoriasis presents with well-circumscribed raised papules and plaques covered with a thick silvery scale. The lesions are located on the scalp, trunk, and extensor areas of the extremities (elbows and knees). (Choice E) Seborrheic keratoses occur in elderly individuals and present with "stuck-on", deeply pigmented or flesh-colored lesions with a velvety or "greasy" surface. Educational objective: Actinic keratosis (AK) develops on chronically sun-exposed areas of the skin in predisposed individuals. The lesions consist of erythematous papules with a central scale and a rough "sandpaper-like" texture. AKs are considered premalignant lesions and have the potential to progress to squamous cell carcinoma.

A 42-year-old man comes to the emergency department due to fever, abdominal pain, and explosive diarrhea. He has a history of HIV but has not been taking any medications. BMI is 18.2 kg/m2. Abdominal examination is notable for right-sided tenderness to palpation. Laboratory results show a CD4 cell count of 38/mm3, leukopenia, and thrombocytopenia. A CT scan of the abdomen reveals colitis. Colonoscopy reveals areas of erosions and ulcerations; pathology demonstrates cells with viral intranuclear and intracytoplasmic inclusions. Antiviral therapy is initiated to treat the colitis with an intravenous agent that does not require intracellular activation and is known to bind in vitro with viral-encoded enzymes such as DNA polymerase, RNA polymerase, and reverse transcriptase. Which of the following agents did this patient most likely receive? Acyclovir Foscarnet Ganciclovir Lamivudine Oseltamivir Saquinavir Sofosbuvir

This patient with HIV likely has cytomegalovirus (CMV) colitis (intranuclear and intracytoplasmic inclusions), which, in some patients (eg, with ganciclovir resistance or refractory thrombocytopenia), is treated with foscarnet. Foscarnet is a pyrophosphate analog that does not require intracellular activation. It directly inhibits both DNA polymerase in herpesvirus and reverse transcriptase in HIV. Foscarnet must be administered intravenously. In addition to treating ganciclovir-resistant CMV infections, foscarnet is useful in patients who have acyclovir-resistant herpesvirus infections. Ganciclovir, a guanine nucleoside analogue structurally similar to acyclovir, is often used for CMV infections. However, it requires intracellular conversion by a virally encoded kinase and then by cellular kinases (Choice C). (Choice A) Acyclovir is a nucleoside analog that must be converted intracellularly to its monophosphate form by a virally encoded thymidine kinase. Cellular kinases convert the monophosphate form into a triphosphate form, which inhibits herpesvirus DNA polymerase-mediated replication. (Choice D) Lamivudine is a cytosine analog classified as a nucleoside reverse transcriptase inhibitor. It must be phosphorylated to its active form, lamivudine triphosphate, by intracellular kinases. Lamivudine inhibits HIV reverse transcription through viral DNA chain termination. (Choices E and F) Oseltamivir and saquinavir do not require intracellular activation but are not used for CMV infection. Oseltamivir is a sialic acid analogue inhibitor of influenza A and B virus neuraminidase. Saquinavir is an HIV protease inhibitor that prevents the cleavage of polyprotein precursors necessary for the generation of functional viral proteins. (Choice G) Sofosbuvir inhibits nonstructural protein 5B (NS5B), an RNA-dependent RNA polymerase needed for hepatitis C virus replication. It requires intracellular activation. Educational objective: Of all the antiviral agents that bind and inhibit DNA polymerase in herpesvirus and reverse transcriptase in HIV, the pyrophosphate analog foscarnet is one of few that do not require intracellular activation by viral or cellular kinases.

A 46-year-old man comes to the office with the skin lesions shown in the photograph below. The patient first noticed the lesions a week ago after he returned from a trip to Hawaii. He has not had any other significant rashes or skin lesions but does have a history of autoimmune thyroiditis. ID=1110 Hypopigmented spots Which of the following would most likely be seen on histopathologic examination of this patient's lesions? Absence of melanocytes in the epidermis Linear pattern of melanocyte proliferation Melanosome aggregates within the cytoplasm of melanocytes Normal melanocytes with diminished pigment transfer to keratinocytes Poor melanin formation in melanocytes

This patient has vitiligo, which is characterized by flat, well-circumscribed patches of depigmentation of variable size due to the partial or complete loss of melanocytes. These lesions often manifest by the second or third decade of life and can be more obvious in individuals with darkly pigmented skin. Vitiligo classically erupts on the face, extremities, axillae, groin, and over hard bony surfaces (eg, knees). Histologic examination of the affected epidermis would demonstrate the absence of melanocytes and melanin pigment. The pathogenesis of melanocyte loss in vitiligo is likely a combination of autoimmune activity, neurohumoral toxicity specific for melanocytes, and melanocytic self-destruction (eg, toxic intermediate exposure during melanin synthesis). Autoimmune activity seems to have the strongest correlation with vitiligo due to the presence of circulating melanocyte antibodies in patients and the high prevalence of concurrent autoimmune disorders (e.g., type I diabetes mellitus, pernicious anemia, Addison disease, autoimmune thyroiditis). (Choice B) Lentigines are small, benign pigmented lesions characterized by linear melanocytic hyperplasia. (Choice C) Café-au-lait spots demonstrate increased melanosome aggregates within the melanocyte cytoplasm. These macules are hyperpigmented and associated with neurofibromatosis type 1. (Choice D) Postinflammatory hypopigmentation can occur after significant inflammatory skin lesions (eg, discoid lupus, psoriasis) and is characterized by redistribution of existing melanin within the skin and/or reduced transfer of melanin to keratinocytes. (Choice E) Individuals with albinism have melanocytes that do not produce melanin because of absent or defective tyrosinase. Educational objective: Vitiligo is a common condition characterized by the loss of epidermal melanocytes. It occurs more commonly in patients with autoimmune disorders (eg, autoimmune thyroiditis, type I diabetes) and results in well-defined, variably sized patches of hypopigmentation.

A 12-year-old boy is brought to the office due to a week of nonproductive cough and rash. He has also had low-grade fevers, malaise, and headaches. The patient has no prior medical conditions and is up to date with immunizations. Lung examination reveals bilateral crackles and wheezes. There is a diffuse rash on his torso and extremities as shown in the exhibit. Which of the following is the most likely cause of this patient's skin lesions? ID=15695 Cytotoxic T-cell-mediated epithelial injury Inflammatory reaction in subcutaneous fat Keratinocyte hyperproliferation and parakeratosis Subepidermal immunoglobulin A deposition Vasoactive mediator release from mast cells

This patient's target-shaped skin lesions likely indicate erythema multiforme (EM), an acute, inflammatory skin disorder that usually develops in the setting of an ongoing infection. Although herpes simplex virus is the most common trigger, respiratory tract infections due to Mycoplasma pneumoniae (eg, low-grade fever, headache, fatigue, cough, wheezes) frequently cause EM in children. EM develops when circulating pathogens are phagocytosed by peripheral mononuclear cells and are brought to the epidermis, where DNA fragments are transferred to keratinocytes via direct cell-to-cell spread (facilitated by upregulation of adhesion molecules). Pathogen-specific cytotoxic T-cells then recognize foreign antigens produced by keratinocytes and initiate an inflammatory cascade that results in epithelial damage. Manifestations of EM include target-shaped lesions on the extremities and trunk that have a central dusky area, a surrounding pale ring of edema, and a peripheral halo of erythema. Mucosal lesions also sometimes occur. (Choice B) Erythema nodosum is a delayed-type hypersensitivity reaction that causes inflammatory nodules in the subcutaneous fat. Although it can be triggered by infection, patients generally have tender, erythematous nodules on the bilateral shins (not target-shaped lesions). (Choice C) Psoriasis is an immune-mediated inflammatory disease associated with cytokine-driven keratinocyte hyperproliferation and parakeratosis. It usually causes erythematous, chronic skin plaques with silvery scales on the extensor elbows, scalp, and/or knees. (Choice D) Dermatitis herpetiformis, a manifestation of gluten sensitivity, is caused by the subepidermal deposition of IgA. It is associated with pruritic papules and vesicles on the forearms, knees, and/or scalp (not target-shaped lesions). (Choice E) Urticaria is caused by the release of inflammatory and vasoactive mediators from mast cells in the superficial dermis. Infections (eg, parasites) can cause urticaria, but patients generally have well-circumscribed, raised, pruritic, erythematous plaques. Sometimes the lesions have central pallor, but they are not target-shaped. Educational objective: Erythema multiforme is a target-shaped, inflammatory skin lesion that typically arises in the setting of infection, particularly with herpes simplex virus or Mycoplasma pneumoniae. It is caused by the deposition of infectious antigens in keratinocytes, leading to a strong cell-mediated (eg, cytotoxic T-cell) immune response.

A 65-year-old man comes to the office due to fatigue, decreased appetite, and weight loss. He has lost 5.4 kg (12 lb) over the last 3 months and eats only small portions of food because he feels full soon after he begins eating. The patient smokes a pack of cigarettes per day and drinks alcohol socially. Blood pressure is 152/90 mm Hg and pulse is 90/min. Physical examination shows a firm, 3-cm lymph node above the left clavicle. Numerous skin lesions are seen on the patient's trunk, as shown below; the lesions were not present during his last physical examination 1 year ago. ID=7649 Which of the following is the most likely diagnosis of this patient's skin lesions? Acanthosis nigricans Malignant melanoma Neurofibromatosis Psoriasis Seborrheic keratosis Tuberous sclerosis

This patient's weight loss, early satiety, positive smoking history, and left supraclavicular adenopathy are concerning for gastric adenocarcinoma (GA). Patients with GA can also have epigastric pain, hematemesis, and occult blood loss in the stool. Physical examination findings in advanced (metastatic) disease include left supraclavicular (Virchow node) or left axillary lymphadenopathy, or a periumbilical mass (Sister Mary Joseph nodule). In addition, rapid onset of numerous seborrheic keratoses (pigmented macules or plaques with a greasy surface and well-demarcated borders) is a classic indicator of internal malignancy (Leser-Trélat sign), particularly GA, and is thought to occur in response to increased tumor production of cytokines or growth factors (eg, IGF-1, fibroblast growth factor). Acanthosis nigricans (a hyperpigmented skin disorder typically associated with insulin resistance) can also occur in malignancy due to a similar mechanism (Choice A). (Choice B) Melanoma characteristically presents as an asymmetric, pigmented lesion with an irregular border, variable coloration, and change in size and appearance over time. Lesions that are multifocal, monotonous, and otherwise asymptomatic are more typical of seborrheic keratoses. (Choice C) Cutaneous neurofibromas are soft, mobile tumors that are occasionally hyperpigmented. They are often solitary and sporadic, although numerous lesions may occur in neurofibromatosis types 1 and 2. However, these inherited disorders typically present in childhood or young adulthood, and type 1 (the more common type) is also characterized by axillary freckling, café-au-lait macules, and iris hamartomas. (Choice D) Psoriasis is an inflammatory dermatosis characterized by well-demarcated plaques with silvery scales. It can affect the trunk but is more common on extensor surfaces of the extremities. Explosive onset of psoriasis is associated with HIV infection. (Choice F) Tuberous sclerosis is an inherited neurocutaneous syndrome. Cutaneous findings include hypopigmented macules, periungual fibromas, connective tissue nevi, forehead fibrous plaques, and facial angiofibromas. Educational objective: Seborrheic keratoses are pigmented macules or plaques with a greasy surface and well-demarcated borders. Rapid onset of numerous lesions is an indicator of internal malignancy (Leser-Trélat sign), especially gastric adenocarcinoma.

A 35-year-old woman comes to the emergency department due to an itchy rash that appeared earlier this morning. She recently returned from a family vacation at a resort in Cancun. Two days ago, the patient's husband developed similar symptoms. The patient has no significant past medical history and takes no medications. She has no known allergies. Her temperature is 37.6 C (99.7 F). On examination, there is a diffuse, pruritic, papulopustular rash that is most noticeable on the trunk and extremities. Microbiologic analysis of a pustular fluid sample demonstrates oxidase-positive gram-negative rods that produce pigment on culture medium. Which of the following is the most likely source of this patient's infection? Food Human contact Insects Pets Pool water Soil

The clinical syndrome described is consistent with "hot tub folliculitis," a superficial pseudomonal infection of the hair follicle. This condition, characterized by a pruritic, papulopustular rash, is most commonly seen with outbreaks from public or hotel swimming pools or hot tubs where the chemicals in the pool water have not been maintained at appropriate concentrations, thereby allowing Pseudomonas aeruginosa proliferation. P aeruginosa is ubiquitous in nature. Many P aeruginosa infections often begin with exposure to a water source or creation of a moist environment (ie, swimmer's ear, hot tub folliculitis, burn wound). P aeruginosa is a motile gram-negative rod that is oxidase-positive, produces green pigment (pyocyanin, pyoverdin) during culture, and is nearly always implicated in "hot tub folliculitis." (Choices A, B, and C) Scabies can be transmitted by human contact but is due to a mite. P aeruginosa is not transmitted by food or insects. (Choice D) Cat scratch disease is due to the oxidase-negative gram-negative Bartonella henselae and can present as vesicular, erythematous, or papular lesions localized to the site of primary inoculation. (Choice F) Although P aeruginosa grows in soil, the most likely source of this patient's infection is exposure to water. Educational objective: "Hot tub folliculitis" is a superficial and self-limited Pseudomonas aeruginosa infection of the hair follicles that tends to occur in minor outbreaks following exposure to a pool or spa in which the chemicals have not been maintained at appropriate concentrations. Pseudomonas are gram-negative, oxidase-positive, nonlactose-fermenting, motile rods that produce green pigment.

A 65-year-old man is admitted to the intensive care unit with second- and third-degree burns over 25% of his total body surface area. Intravenous fluids and analgesics are administered, and his wounds are cleaned and debrided. After 7 days, the patient develops a fever and his leukocyte count rises to 16,000 cells/µL. Examination of the burns on his left leg shows surrounding warmth, erythema, and induration. Gram stain of his wound tissue is shown in the image below. ID=8858 Culture on MacConkey agar grows oxidase-positive colonies that do not ferment lactose. Which of the following is the best initial treatment for this patient? Cefepime Ceftriaxone Doxycycline Moxifloxacin Trimethoprim-sulfamethoxazole

Burn patients are at increased risk of bacterial infection due to loss of the barrier function of the skin, post-burn immune dysfunction, and lack of blood flow to necrotic areas. Common pathogens include Gram-positive (eg, Staphylococcus aureus, Enterococcus species) and Gram-negative (eg, Pseudomonas aeruginosa, Escherichia coli, Klebsiella pneumoniae, and Acinetobacter species) bacteria. This patient's burn is infected with Gram-negative rods that are oxidase-positive and non-lactose- fermenting, making P aeruginosa the most likely pathogen. Cephalosporins such as cefepime and ceftazidime have good anti-pseudomonal coverage. (Choice B) Ceftriaxone is a third-generation cephalosporin used primarily for the treatment of community-acquired pneumonia and bacterial meningitis. It has activity against some Gram-positive (eg, Streptococcus pneumoniae, S pyogenes) and Gram-negative (eg, Haemophilus influenzae, K pneumoniae, E coli) pathogens. Unlike ceftazidime, it has poor activity against P aeruginosa. (Choice C) Doxycycline is effective against Lyme disease and many tickborne rickettsial diseases but does not have good anti-pseudomonal coverage. (Choice D) Moxifloxacin is a fluoroquinolone used mainly for the treatment of bacterial infections that cause exacerbation of chronic obstructive pulmonary disease, sinusitis, and community-acquired pneumonia. Unlike ciprofloxacin and levofloxacin, moxifloxacin has poor anti-pseudomonal activity. (Choice E) Trimethoprim-sulfamethoxazole is used to treat susceptible organisms that cause urinary tract, gastrointestinal, and skin infections. It is the drug of choice for organisms such as Pneumocystis jiroveci, Toxoplasma gondii, Nocardia, and Stenotrophomonas maltophilia. It does not have activity against P aeruginosa. Educational objective: Pseudomonas aeruginosa is a major pathogen in burn patients. Only a few specific penicillins (eg, ticarcillin, piperacillin) and cephalosporins (eg, ceftazidime, cefepime) have activity against it. Certain aminoglycosides, fluoroquinolones (eg, ciprofloxacin, levofloxacin), and carbapenems (eg, imipenem, meropenem) are also effective.

A 9-month-old girl is brought to the emergency department after a 2-minute generalized seizure. The patient has had a fever for the past 12 hours and has been very fussy. She takes no medications and has no allergies. Her temperature is 39.4 C (103 F). Examination shows no abnormalities, and the patient is discharged home. Three days later, she is afebrile but develops a maculopapular rash on her trunk. Infection with which of the following is the most likely cause of this patient's condition? Coxsackievirus A16 Herpes simplex virus type 1 Human herpesvirus 6 Measles virus Parvovirus B19 Rubella virus Streptococcus pyogenes

ID=11822 This patient's presentation is consistent with roseola infantum, which is most commonly caused by human herpesvirus 6 (HHV-6). Febrile seizures can be caused by any febrile illness, and HHV-6 is one of the most common causes. However, most patients with HHV-6 infection do not develop seizures. Primary symptoms include high fever for 3-5 days followed by an erythematous maculopapular rash. The rash usually starts on the trunk and spreads to the face and extremities. The diagnosis is based on clinical presentation, and the infection is typically benign and self-limited. (Choice A) Coxsackievirus A16 is a common cause of hand-foot-mouth disease in children. Patients typically have erythematous oropharyngeal ulcers and maculopapular or vesicular lesions on the palms and soles. (Choice B) Children with primary infection with herpes-simplex virus type 1 (HSV-1) have painful gingivostomatitis, often accompanied by fever and irritability. Neonatal HSV, most commonly caused by HSV-2, can present with fever and seizures in the newborn period. (Choices D and F) Rubella and measles are vaccine-preventable viral infections that typically present with a maculopapular rash on the face that then spreads to the trunk and extremities. Both infections can be distinguished from roseola by the pattern of rash development. In addition, patients with rubella often have Forchheimer spots on the soft palate, and measles is characterized by pathognomonic Koplik spots on buccal mucosa. (Choice E) Parvovirus B19 causes erythema infectiosum, or fifth disease, which can present as a flulike illness followed by a characteristic "slapped cheek" rash and truncal reticular rash. Parvovirus B19 can be associated with an aplastic crisis in those with pre-existing hematologic abnormalities (eg, sickle cell anemia). (Choice G) Streptococcus pyogenes (Group A Streptococcus) can cause acute, exudative pharyngitis that is associated with fever, headache, and abdominal pain in school-aged children. It can also be accompanied by scarlet fever, a diffuse "sandpaper"-textured rash most prominent in skinfolds (eg, groin, axillae, antecubital fossa). Educational objective: Human herpesvirus 6 infection (roseola) classically presents with fever for 3-5 days followed by a truncal rash. It is also the most common cause of febrile seizures.

A 24-year-old woman comes to the office for a preemployment medical evaluation. The patient has no known medical problems but reports that her skin bruises and scars easily. She says that most of her family members have a very "flexible" body, and her brother works in a circus as a contortionist. The patient takes no medications and has no allergies. She does not use tobacco, alcohol, or drugs. Physical examination findings are shown in the exhibit. This patient most likely has an inherited defect in which of the following proteins? Collagen Elastin Fibrillin-1 Hyaluronic acid Laminin Proteoglycan

ID=1244 The extracellular matrix is a network of interstitial proteins that maintain normal tissue architecture. Collagen is a major component of connective tissue and consists of 3 polypeptide α chains held together by hydrogen bonds to form a ropelike triple helix structure (tropocollagen). Lysyl oxidase then forms covalent bonds between individual tropocollagen molecules, generating mature collagen fibers. The variation in amino acid sequences in the collagen α chains gives rise to collagen diversity in different tissues. Collagen types I, II, III, and V provide tensile strength in skin, bones, cartilage, tendons, and blood vessels. Ehlers-Danlos syndrome (EDS) is a group of hereditary disorders involving a defect in collagen synthesis. EDS usually manifests clinically as hypermobile joints, overelastic skin, and fragile tissue susceptible to bruising, wounds, and hemarthrosis. Common mutations leading to EDS phenotypes include deficiencies of the lysyl hydroxylase and procollagen peptidase enzymes responsible for collagen synthesis. (Choice B) Elastin, a fibrous protein in the connective tissue, is named for the elastic properties it imparts to skin, blood vessels, and lung alveoli. Elastin fibers can be stretched to several times their original length but will recoil when the stretching forces are withdrawn. Elastin is synthesized from the polypeptide precursor tropoelastin. (Choice C) Fibrillin-1 is a major component of the microfibrils that form a sheath around elastin. Microfibrils are abundantly present in blood vessels and in the suspensory ligaments of the lens. Defects in the fibrillin-1 gene cause classic autosomal dominant Marfan syndrome. (Choice D) Hyaluronic acid is another major component of the soft tissue's extracellular matrix, including synovial fluid and skin. Exogenous injection can be used to restore viscoelasticity to the synovial fluid in osteoarthritis; soft-tissue fillers can also be used in patients concerned about age-related volume loss (eg, nasolabial folds). (Choice E) Laminins are heterotrimeric glycoproteins that bind to type IV collagen underlying epithelial cells. They contribute to the organization and function of the basal lamina (basement membrane). (Choice F) Proteoglycans are composed of glycosaminoglycans (GAGs), which provide compressibility to tissues. Patients with deficiencies in lysosomal enzymes cannot break down GAGs, resulting in mucopolysaccharidoses (eg, Hurler syndrome, Hunter syndrome) characterized by soft tissue and skeletal disease. Educational objective: Ehlers-Danlos syndrome (EDS) is a heritable connective tissue disease associated with abnormal collagen formation. EDS usually manifests clinically as overflexible (hypermobile) joints, overelastic (hyperelastic) skin, and fragile tissue susceptible to bruising, wounding, and hemarthrosis.

A 2-year-old boy is evaluated for easy bruising. His parents report that he develops marked bruising and open wounds following minor trauma. The skin is difficult to suture due to its extreme fragility. Physical examination reveals hyperextensible skin, multiple ecchymoses over the forearms and pretibial regions, and an umbilical hernia. A skin biopsy is performed, and histochemical evaluation of the biopsy reveals a defect in extracellular processing of collagen. Which of the following steps of collagen synthesis is most likely impaired in this patient? Glycosylation of hydroxylysine residues Interchain C-terminal disulfide bone formation N-terminal propeptide removal Proline residue hydroxylation Triple helix formation

ID=1245 This child likely has Ehlers-Danlos syndrome, a group of rare hereditary disorders characterized by defective collagen synthesis. The condition can be caused by a deficiency in procollagen peptidase, the enzyme that cleaves terminal propeptides from procollagen in the extracellular space. Impaired propeptide removal results in the formation of soluble collagen that does not properly crosslink. Consequently, patients often have joint laxity, hyperextensible skin, fragile tissue with easy bruising, and poor wound healing. Each collagen molecule consists of 3 polypeptide α-chains held together by hydrogen bonds, forming a triple helix. Collagen assumes this conformation because each of the α-chains has a simple, repetitive amino acid sequence represented as (Gly-X-Y)n. The smallest amino acid, glycine (Gly), is necessary at every third position to ensure compact coiling of the helix. Many of the amino acids represented by X and Y are proline residues, which kink the polypeptide chain and enhance the rigidity of the helical structure due to their ring configuration. Mature collagen is synthesized by fibroblasts, osteoblasts, and chondroblasts through the following steps: As translation begins in the cytoplasm, an amino acid signal sequence at the N-terminus of the α-chain facilitates ribosomal binding to the rough endoplasmic reticulum (RER) and passage of the growing polypeptide chain (pre-pro-α-chain) into the RER. Inside the RER, the hydrophobic signal sequence is cleaved to yield the pro-α-chain. Proline and lysine at the Y positions of the pro-α-chain are hydroxylated to hydroxyproline and hydroxylysine, respectively (Choice D). Glycosylation of select hydroxylysine residues also occurs within the RER (Choice A). The central helical region of the pro-α-chain is flanked by N- and C-terminal propeptides. Disulfide bond formation between the C-terminal propeptide region of 3 α-chains brings the chains into an alignment favorable for assembly into a triple helix (procollagen molecule) (Choices B and E). Procollagen molecules are then transported through the Golgi apparatus into the extracellular space. The N- and C-terminal propeptides are cleaved by procollagen peptidases, converting procollagen into less soluble tropocollagen. Tropocollagen monomers self-assemble into collagen fibrils. Finally, lysyl oxidase helps create covalent crosslinks between collagen fibrils to form strong collagen fibers. Educational objective: Ehlers-Danlos syndrome is a group of rare hereditary disorders characterized by defective collagen synthesis. It can be caused by procollagen peptidase deficiency, which results in impaired cleavage of terminal propeptides in the extracellular space. Patients often have joint laxity, hyperextensible skin, and tissue fragility due to the formation of soluble collagen that does not properly crosslink.

An 18-year-old man comes to the urgent care clinic due to painful erythema affecting his extremities, trunk, and face. He is vacationing in Florida and spent 5 hours at a Gulf coast beach earlier in the day. The patient did not apply any sunscreen as it was cloudy. The physician explains that cloud cover does not afford a high degree of protection against the sun, especially with prolonged exposure in highly reflective environments like the beach. He is advised to wear protective clothing and apply sunscreen to prevent recurrence. Which of the following is most likely to happen within the patient's skin cells as a result of his exposure? Covalent bond formation between adjacent purine bases End-joining repair of double-stranded DNA breaks Endonuclease nicking of the damaged DNA strand Hypermethylation of residues in the undamaged DNA strand Removal of deaminated bases by glycosylase

ID=1477 DNA can be damaged by a number of agents, including chemicals, ultraviolet radiation, and ionizing radiation. Ultraviolet rays damage DNA primarily through formation of abnormal covalent bonds between adjacent thymine or cytosine residues (pyrimidine dimers). The presence of pyrimidine dimers interferes with base recognition during transcription and replication, and DNA mutations can result if the damage is not repaired. Pyrimidine dimers are removed by nucleotide excision repair. In this process, a specific endonuclease complex detects abnormalities in the DNA structure caused by the formation of DNA photoproducts. The endonuclease complex then nicks the damaged strand on both sides of the pyrimidine dimer, and the defective region is excised. DNA polymerase synthesizes new DNA in the place of the damaged DNA, and DNA ligase seals the final remaining nick. Mutations that impair the components involved in nucleotide excision repair cause xeroderma pigmentosum, a condition characterized by severe photosensitivity and the development of skin cancers at a young age. (Choice A) Ultraviolet radiation causes the formation of pyrimidine-pyrimidine dimers, not purine-purine dimers. (Choice B) Exposure to ionizing radiation (x-rays and gamma rays) causes double-strand DNA breaks. The fractured ends can be joined by nonhomologous end joining. (Choice D) Incorrect base substitution occurs during normal DNA replication and can result in mutations if they are not corrected. DNA mismatch repair in certain prokaryotes is guided by hypermethylation of the parent strand, which helps to identify the non-mutated strand for use as a template. (Choice E) Deamination of DNA bases (eg, cytosine conversion to uracil, adenine to hypoxanthine) can occur spontaneously or secondary to chemical exposure. These errors are corrected by base excision repair. In this process, abnormal bases are recognized and removed by specific glycosylases without disruption of the phosphodiester backbone. The apurinic and apyrimidinic residues are then removed by specific endonucleases and replaced with the correct base by DNA polymerase. Educational objective: Pyrimidine dimers are formed in DNA as a result of ultraviolet light exposure. They are recognized by a specific endonuclease complex that initiates the process of repair by nicking the damaged strand on both sides of the pyrimidine dimer. The damaged segment is then excised, and replacement DNA is synthesized by DNA polymerase.

A 24-year-old woman comes to the office due to worsening acne. Over the past 3 months, the patient has had frequent breakouts on the shoulders and upper back. She had acne on her face during puberty, which cleared spontaneously after several years, but has never before had symptoms on her trunk. The patient has no other medical conditions, and her only medication is a combination oral contraceptive. She has been training for a half-marathon for the past 4 months and has lost 6.8 kg (15 lb) during this period. She does not use tobacco, alcohol, or illicit drugs, and she relocated from California to Wisconsin for work a year ago. Vital signs are within normal limits. Physical examination shows acneiform eruptions on the shoulders and mid-back. Which of the following is most likely contributing to worsening of this patient's acne? Exposure to colder temperatures Exposure to contaminated water Rapid loss of body weight Use of hormonal contraceptive Use of tight-fitting sports clothing

ID=15373 Acne vulgaris is a disorder of the pilosebaceous follicle. Hyperkeratinization and obstruction of the follicle in combination with glandular enlargement and increased sebum production provides an optimal environment for Cutibacterium (Propionibacterium) acnes. This organism metabolizes sebaceous lipids, releasing inflammatory factors that lead to follicular and perifollicular inflammation. Acne is most common in areas with a high concentration of sebaceous follicles, including the face, back, chest, and upper arms. Androgens stimulate production of sebum, and increased androgen levels (eg, puberty, polycystic ovary syndrome) increase the incidence of acne. Obstruction of the glands, such as by oil-based hair products (pomade acne), also promotes acne. In addition, mechanical irritation can promote acne by increasing glandular occlusion and disruption of follicles (acne mechanica). Sports participation, as in this patient, frequently triggers acne due to the use of tight-fitting clothing (eg, sports bras) and protective gear (eg, helmets). (Choice A) High ambient temperatures can worsen acne, leading to formation of painful nodules on the trunk (tropical acne). Colder temperatures are not associated with acne. (Choice B) Exposure to water contaminated with Pseudomonas aeruginosa can cause erythematous papules and pustules around hair follicles (hot-tub folliculitis). This condition begins acutely, resolves spontaneously within 1-2 weeks, and typically involves the skin under a bathing suit (eg, buttocks) or on the extremities. (Choice C) Obesity is associated with an increased risk of acne, but weight loss generally does not worsen acne. (Choice D) Typical oral contraceptives do not promote acne. Rather, many commonly used contraceptives can reduce acne due to decreased synthesis of androgens. Educational objective: Risk factors for acne include increased androgen levels; obstruction of pilosebaceous glands by oil-based hair products; and mechanical irritation of skin follicles. Sports participation frequently triggers acne due to the use of tight-fitting clothing and protective gear.

A 22-year-old woman comes to the emergency department with dysuria, vulvar pain, and pruritus. The patient has also felt feverish and fatigued for the last few days. She has a new sexual partner and has been using a hormonal vaginal ring for contraception. Physical examination reveals bilateral inguinal lymphadenopathy and tender vesicular lesions covering the labia majora and perineum. Which of the following conditions will most likely result as a sequela of this patient's infection? Fluctuant lymphadenitis Locomotor ataxia Postherpetic neuralgia Recurrent genital ulcers Squamous cell vulvar carcinoma

ID=1549 This patient with tender, vesicular genital lesions and systemic symptoms (eg, fever, fatigue) likely has a primary genital herpes infection. Patients typically develop a characteristic genital rash consisting of small, grouped vesicles that progress into ulcers with crusting; the lesions may be pruritic and associated with painful inguinal lymphadenopathy. Genital herpes is caused by herpes simplex virus (HSV) type 1 or 2; patients exposed to one serotype can acquire the other. HSV can lie dormant in the sacral dorsal root ganglia and then reactivate; as a result, patients develop recurrent genital lesions. Recurrence is more common with HSV-2, and tends to be localized (no systemic symptoms) or asymptomatic (thereby increasing the risk for inadvertent transmission). Barrier contraception use decreases transmission. (Choice A) Fluctuant lymphadenitis (pus-filled inguinal buboes) can occur 1-2 weeks after Haemophilus ducreyi infection (ie, chancroid). Chancroid presents with multiple, painful genital ulcers and painful lymphadenopathy, but the lesions are typically deep with a gray exudate. Systemic symptoms are uncommon. (Choice B) Untreated syphilis (Treponema pallidum infection) can infect the dorsal roots of the spinal column and lead to tertiary neurosyphilis, which may be characterized by tabes dorsales (locomotor ataxia) and general paresis. Unlike primary genital herpes, primary syphilis typically presents with a painless genital ulcer with a clean base (chancre). (Choice C) Varicella-zoster virus also invades the dorsal root sensory ganglia, and reactivation causes herpes zoster (shingles). One complication is postherpetic neuralgia (persistent pain after resolution of herpes zoster lesions). Patients typically have a painful vesicular rash in a dermatomal distribution and no systemic involvement. (Choice E) Human papillomavirus (HPV) infection increases the risk of cervical and vulvar carcinoma. Certain strains (HPV 6 and 11) can cause anogenital warts (condylomata acuminata), which are pink or skin-colored papules or verrucous lesions, not ulcers. In addition, primary HPV infections do not cause fever or painful lymphadenopathy. Educational objective: Primary genital herpes (herpes simplex virus type 1 or 2 infection) presents as multiple, grouped vesicles on the genitals that progress to ulcers. Primary infections can cause systemic symptoms (eg, fever, malaise) and painful inguinal lymphadenopathy. Because HSV can invade and lie latent in the sacral dorsal root ganglia, patients are at risk for viral reactivation and recurrent genital lesions.

A 26-year-old man is brought to the emergency department due to fever and lethargy. His girlfriend says the patient abruptly began experiencing fever, chills, vomiting, and diarrhea several hours ago, which were quickly followed by lightheadedness and lethargy. He has no prior medical conditions other than an episode of epistaxis after a bar fight 3 days ago. The patient does not smoke cigarettes or use injection drugs. There is no history of exposure to sick contacts, and he has not eaten anything out of the ordinary. Temperature is 38.9 C (102 F), blood pressure is 90/60 mm Hg, and pulse is 120/min. Physical examination shows a diffuse, erythematous rash. There is an anterior nasal packing in the left nostril, removal of which shows mild mucosal erythema with a purulent discharge. Cardiopulmonary and abdominal examinations reveal no abnormalities, and signs of meningeal irritation are absent. Which of the following processes is most essential in pathogenesis of this patient's current condition? Activation of toll-like receptors of bacterial components Binding of T-cell receptors by bacterial secretory products Bridging of cell-bound IgE by multivalent antigens Phagocytosis of bacteria in the blood by neutrophils Presentation of processed antigens by dendritic cells

ID=15509 Staphylococcal toxic shock syndrome (TSS) is an exotoxin-mediated disease associated with rapid-onset high fever, hypotension, multiorgan failure, and a diffuse, erythematous rash. Most cases are linked to prolonged tampon use or nasal wound packing, which provides a medium for localized Staphylococcus aureus proliferation and the elaboration of pyrogenic toxin superantigens (eg, toxic shock syndrome toxin-1 [TSST-1]) into the bloodstream. Pyrogenic toxin superantigens are generated by strains of S aureus (and Streptococcus pyogenes) that have acquired an underlying mobile genetic element via bacteriophage or plasmid. These exotoxins are called superantigens because they bind to the invariant region of the MHC-II complex of antigen-presenting cells without first being internalized and processed (MHC-II antigens are normally processed by the phagolysosome) (Choice E). The superantigen/MHC-II complex subsequently interacts with the variable part of the T-cell receptor beta chain, which results in the nonspecific stimulation of a large percentage (>20%) of total T cells and the massive release of inflammatory cytokines (eg, IL-1, IL-2, TNF-alpha & beta, interferon-gamma). Cytokine release mediates the major manifestations of TSS including fever, increased capillary permeability, and hypotension. (Choice A) Endotoxin in the cell membrane of gram-negative bacteria can cause rapid-onset fever and hypotension due to activation of toll-like receptors on cells of the innate immune system. However, in this case, the presence of nasal packing and a diffuse, erythematous rash make TSS far more likely than gram-negative sepsis. (Choice C) Anaphylactic shock occurs due to antigen-stimulated cross-linking of IgE that is bound to basophils or mast cells, which activates the cell and releases massive amounts of inflammatory mediators. Anaphylactic shock is typically characterized by hypotension, bronchospasm, and shortness of breath; most cases are triggered by insect stings, food, or medications, and manifestations occur shortly after exposure. (Choice D) Patients who develop TSS usually have a noninvasive infection in the vaginal canal or area of wound packing. TSS is mediated by the absorption of superantigens into the blood, not the invasion of bacteria. Educational objective: Toxic shock syndrome is typically associated with the prolonged use of tampons or wound packing, which allows Staphylococcus aureus to replicate locally and release pyrogenic toxic superantigens (eg, toxic shock syndrome toxin-1) into the blood. Superantigens bind to the MHC-II complex of antigen-presenting cells without processing and nonspecifically activate T cells. This leads to a dramatic release of inflammatory cytokines, which causes the manifestations of the disease (eg, hypotension; high fever; organ failure; diffuse, erythematous rash).

A 29-year-old woman comes to the office with malaise, fever, and extremely painful genital lesions. She has never had such symptoms before. She is sexually active with a new partner. On examination, the patient has several tiny, thin-walled vesicles on an erythematous base and exquisitely tender, shallow ulcers on the labia. Tender inguinal lymphadenopathy is present. Tzanck smear of the genital lesions shows multinucleated giant cells. A drug is prescribed that can be used to manage this acute episode and prevent future episodes of this patient's condition. This drug most likely works by which of the following mechanisms? Incorporation into newly replicating viral DNA Inhibition of viral protease activity Inhibition of virion release from infected cells Prevention of viral entry into the target cells Upregulation of interferon synthesis in infected cells

ID=1551 This patient's new genital rash and positive Tzanck smear are consistent with a primary genital herpes simplex virus (HSV) infection, likely due to HSV-2. Antiviral drugs for the treatment of primary genital herpes include acyclovir, valacyclovir (acyclovir prodrug with better oral bioavailability), and famciclovir. Acyclovir treatment during a primary episode can reduce duration of viral shedding, time to lesional healing, constitutional symptoms, and local pain. Chronic daily suppressive therapy (lower dose) can be used in patients who have recurrent episodes. In infected host cells, acyclovir (a nucleoside analog) is converted into acyclovir monophosphate principally via virus-encoded thymidine kinase. Cellular enzymes then convert the monophosphate into acyclovir triphosphate, which competes with deoxyguanosine triphosphate for viral DNA polymerase. When acyclovir triphosphate becomes incorporated into the replicating viral DNA chain, viral DNA synthesis is terminated. Acyclovir does not significantly impact uninfected cells in vitro as uptake into these cells is poor, phosphorylation in the absence of viral thymidine kinase is minimal, and cellular α-DNA polymerase has significantly less affinity for acyclovir triphosphate than does viral DNA polymerase. (Choice B) Inhibition of viral protease activity is the mechanism of action for drugs such as indinavir, an antiretroviral used in HIV treatment. (Choice C) Prevention of virion release from infected cells is the mechanism of action for drugs such as neuraminidase inhibitors (eg, oseltamivir), which are used in the early treatment of influenza. (Choice D) Drugs such as enfuvirtide and maraviroc, antiretrovirals used in HIV treatment, prevent viral entry into the target cells. Docosanol, a topical agent that prevents viral entry, is sometimes used for herpes labialis, not genital HSV. (Choice E) Upregulation of interferon synthesis in infected cells is accomplished by many interleukins. Educational objective: Antiviral drugs currently recommended for the treatment of primary genital herpes include the nucleoside analogs (eg, acyclovir). These are incorporated into newly replicating viral DNA and ultimately terminate viral DNA chain synthesis.

A 61-year-old man comes to the office due to a painful rash on his face for the past 3 days. The patient has no other medical conditions and takes no medications. He is a retired schoolteacher and lives with his wife. Skin examination findings are shown in the exhibit. The remainder of the physical examination shows no abnormalities. Which of the following events most likely immediately preceded the appearance of skin rash in this patient? ID=1552 Alteration in the patient's normal skin flora Close physical contact with an infected person Inhalation of pathogen-containing aerosols Reactivation of latent organism in sensory ganglia Waning of organism-specific antibodies

ID=1552 This patient has a crusted vesicular rash in a dermatomal distribution consistent with shingles (herpes zoster). Shingles is caused by reactivation of varicella-zoster virus (VZV), a double-stranded DNA virus of the herpesvirus family. Primary VZV infection occurs most commonly in childhood. Inoculation in the nasopharyngeal mucosa is followed by migration of the virus to the regional lymphatics. Subsequent dermatotrophic migration produces a diffuse vesicular rash associated with fever, malaise, and pharyngitis (chickenpox). Then, the virus migrates via sensory nerves to cranial nerve and dorsal spinal ganglia, where it lies dormant for decades. Most patients who have had primary chickenpox do not develop recurrent diffuse disease. Over time, however, waning cell-mediated immune function allows reactivation of the virus, which typically spreads down a single spinal nerve to cause a painful, erythematous, vesicular rash in a dermatomal distribution. The primary risk factor for reactivation is increasing age; other factors can include malignancy, autoimmune disease, and immune-suppressing therapy. The varicella vaccine is recommended for prevention of primary disease in children, women of childbearing age, adults with sustained risk of exposure, and household contacts of immunocompromised hosts. Zoster vaccine is recommended for adults age ≥50 and reduces the risk of reactivation. (Choice A) Colonization of the skin by transient flora (eg, Staphylococcus aureus, Candida species, gram-negative bacilli) occasionally leads to clinical disease, especially following antibiotic exposure (eg, vaginal candidiasis). However, VZV does not remain on the surface of the skin for prolonged periods. (Choices B and C) Primary VZV infection (ie, chickenpox) can be acquired by direct contact or airborne aerosol from patients with active chickenpox or zoster lesions. However, primary VZV infection causes diffuse lesions; lesions in a dermatomal distribution are seen only with reactivation from sensory ganglia. (Choice E) Declining VZV-specific immunity increases the risk of reactivation. However, the immune response to VZV is primarily based on cell-mediated, rather than humoral (ie, antibodies), factors. Educational objective :Primary varicella-zoster virus infection (chickenpox) occurs most commonly in childhood. Subsequently, the virus migrates to the sensory ganglia, where it lies dormant for decades. Over time, waning cell-mediated immunity allows reactivation of the virus, which spreads down a single nerve to cause a painful, erythematous, vesicular rash in a dermatomal distribution.

A 25-year-old woman comes to the office due to a 4-week history of a pruritic skin rash. The patient has no chronic medical conditions and takes no medication. Vital signs are normal. On examination, there are pink papules distributed symmetrically over the anterior surfaces of the shins and ankles. Shave biopsy reveals prominent hyperkeratosis along with a thickened granular layer. In addition, there is a bandlike infiltrate of mononuclear cells (predominantly lymphocytes) in the subjacent superficial dermis that involves the overlying epidermis. The rete ridges have a sharpened, sawtooth appearance. The basal layer of the epidermis is degenerated, with scattered colloid bodies. Which of the following is the most likely diagnosis? Atopic dermatitis Contact dematitis Erythema nodosum Lichen planus Psoriasis

ID=16395 This patient has a pruritic rash consistent with lichen planus (LP), an immunologically mediated skin disorder primarily affecting middle-aged adults. The skin lesions typically occur symmetrically on the flexural surfaces of the wrists and ankles but can also involve the nails, oral mucous membranes, and genitalia. The characteristic skin lesions are described by the 5 Ps: pruritic, purple/pink, polygonal papules and plaques and can form along lines of minor trauma (Koebner phenomenon). Chronic lesions often show white, lacy markings known as Wickham striae. Although the etiology of LP is unknown, it is characterized by a (CD8+) T cell-mediated response to the cells along the junction of the dermis and epidermis, resulting in the following characteristic histologic findings: Chronic hyperkeratosis (thickening of stratum corneum) Lymphocytic infiltrates at the dermoepidermal junction (interface dermatitis) Scattered eosinophilic, colloid (Civatte) bodies (apoptotic keratinocytes) Hypergranulosis (thickening of stratum granulosum) Sawtooth rete ridges (Choices A and B) Atopic dermatitis can cause a pruritic rash affecting the extremities, but it usually presents in childhood and most patients have a history of other atopic disorders (eg, allergic rhinitis, asthma). The lesions in contact dermatitis are also pruritic but occur in localized areas corresponding to the specific exposure (eg, waist with nickel belt buckle exposure). Biopsy in these conditions would show spongiosis (epidermal edema) in the acute phase and acanthosis (diffuse epidermal hyperplasia) in chronic disease. (Choice C) Erythema nodosum is a delayed-type hypersensitivity reaction that presents with tender inflammatory nodules in the subcutaneous fat, primarily on the shins. The characteristic histopathologic finding is septal panniculitis in the subcutaneous tissues, with a variable (lymphocytes, neutrophils, macrophages) infiltrate. (Choice E) Psoriasis is an immune-mediated inflammatory disease characterized by cytokine-driven keratinocyte hyperproliferation leading to acanthosis with clubbed rete ridges, parakeratosis (retention of nuclei in stratum corneum), and thinned stratum granulosum. It also demonstrates the Koebner phenomenon but usually presents with chronic erythematous plaques with silvery scales on the extensor surfaces of the knees and elbows, scalp, or neck. Educational objective: Lichen planus presents with pruritic, purple/pink, polygonal papules and plaques that can affect the flexural surfaces of the wrists and ankles, along with the nails, oral mucous membranes, and genitalia. Histologic findings include hyperkeratosis (thickening of stratum corneum), lymphocytic infiltrates at the dermoepidermal junction, hypergranulosis (prominent granular layer), sawtooth rete ridges, and scattered eosinophilic colloid bodies.

The antiviral activity of a guanosine derivative is tested by examining cells infected with different viruses for characteristic abnormal changes in cell morphology due to the viruses. These changes are known as the cytopathic effect (CPE). In the presence of significant antiviral activity, CPE will be inhibited. The following results are obtained from the study: CPE without antiviral agent CPE with antiviral agent Herpes simplex virus 1+++ − Herpes simplex virus 2 +++ − Epstein-Barr virus+++ ++ Varicella zoster virus+++ − Cytomegalovirus+++ ++ A difference in which of the following best explains the variation in susceptibility of the various viruses to this specific antiviral agent? DNA polymerase structure Drug degradation rate Drug phosphorylation rate Viral capsid protein composition Viral protease activity

ID=1645 The drug described in this study demonstrates significant antiviral activity (cytopathic effect) against herpes simplex virus type 1 (HSV 1), HSV 2, and varicella zoster virus (VZV) but has weak activity against Epstein-Barr virus (EBV) and cytomegalovirus (CMV). This agent is most likely acyclovir, a guanosine analog. Once acyclovir enters the HSV-infected host cell, it is phosphorylated to acyclovir monophosphate, principally via a virally-encoded thymidine kinase (TK). This is the rate-limiting step in acyclovir activation. Acyclovir monophosphate is then phosphorylated by cellular enzymes into the active triphosphate form, which impairs viral DNA polymerase-mediated replication of HSV. EBV and CMV do not produce the same TK as HSV and VZV. As a result, EBV- or CMV-infected cells cannot easily convert acyclovir into its pharmacologically active form. At very high intracellular concentrations of acyclovir, some of the drug appears to be activated by unidentified cellular phosphorylating enzymes, which may explain the partial susceptibility of EBV and CMV to higher acyclovir doses. The partial activity of acyclovir against CMV may also be mediated by inhibited synthesis of a virus-specific polypeptide. (Choices A, B, and E) Mutations in viral DNA polymerase can lead to acyclovir resistance, but differences in TK are the best explanation for the observed results. Drug degradation rate and viral protease activity are not responsible for differences in viral susceptibility to acyclovir. (Choice D) TK is an enzyme produced in infected cells, but it is not present in the capsid. Educational objective: Monophosphorylation of acyclovir by a viral thymidine kinase is the first (and rate-limiting) step in conversion of acyclovir to its active triphosphate form. Acyclovir and related drugs (eg, famciclovir, valaciclovir) are more effective against herpes simplex virus and varicella zoster virus than cytomegalovirus and Epstein-Barr virus.

A 4-year-old boy is brought to the office due to high fever, nasal discharge, and cough for 3 days. His family recently returned from a vacation in Mexico. Physical examination shows an ill-appearing, febrile child. The patient has bilateral conjunctival injection and several small white spots with an erythematous base on his buccal mucosa. A review of the medical record shows parental refusal of preventive health services in favor of a "natural" approach to the child's health. Over the next several days, which of the following is most likely to develop? Jaundice Maculopapular rash Parotid swelling Paroxysmal cough Upper airway obstruction Vesicular rash Watery diarrhea

ID=1669 missing ID=1670 Measles virus, or rubeola, is an enveloped, nonsegmented, negative-sense RNA virus of the Paramyxoviridae family. This highly contagious infection is transmitted by airborne or person-to-person contact. Classically, measles presents with a prodrome of fever, cough, rhinorrhea, and conjunctivitis. Koplik spots on the buccal mucosa are a pathognomonic finding and consist of small white, blue, or gray specks on an erythematous base. These symptoms precede a maculopapular exanthem that starts on the face and spreads down the trunk and extremities. The parents' refusal of preventive medicine also supports a diagnosis of measles as the child likely did not receive the recommended combined measles, mumps, and rubella (MMR) vaccine. This live, attenuated vaccine is safe; serious side effects are extremely rare. (Choice A) Hepatitis A spreads through the fecal-oral route and is generally an acute and self-limited disease. Hepatitis B is transmitted via bodily fluids and causes acute and chronic disease. These viruses cause elevated liver enzymes and hyperbilirubinemia (jaundice). (Choice C) Mumps is a member of the Paramyxoviridae family and spreads by respiratory droplets or person-to-person contact. Infection classically causes parotitis, orchitis, and aseptic meningitis. (Choice D) Pertussis is caused by the gram-negative bacteria Bordetella pertussis and spread by respiratory droplets. It is highly contagious and presents with paroxysms of cough and an inspiratory "whoop" sound in children. (Choice E) Epiglottitis is linked with Haemophilus influenzae type b, a gram-negative, encapsulated bacteria spread by respiratory droplets. Patients often present with tripod positioning in an attempt to increase airflow due to life-threatening airway obstruction. (Choice F) A diffuse vesicular rash of different stages is typical of chickenpox, or varicella-zoster virus (VZV) infection. VZV is a double-stranded, enveloped, DNA virus that is part of the Herpesviridae family. Transmission occurs via respiratory droplets or direct contact. (Choice G) Rotavirus, a double-stranded RNA virus of the Reoviridae family, causes profuse watery diarrhea in infants, potentially leading to dehydration and death. Educational objective: Measles presents with fever, cough, rhinorrhea, and conjunctivitis followed by a maculopapular rash that starts on the face and spreads downwards. Koplik spots are pathognomonic for measles and characterized by tiny white or blue-gray lesions on the buccal mucosa.

A 13-year-old boy is brought to the clinic by his parents due to a change in behavior. His special education teacher reported that for the last several weeks he has frequently taken off his shoes to scratch his feet during class. The patient has a history of learning disability but is otherwise healthy. Examination shows erythema and excoriations between the toes of both feet. Microscopy of skin scrapings from the rash is shown in the exhibit. Which of the following antimicrobial drugs is likely to be the most effective against the pathogen causing this patient's condition? ID=104 Albendazole Clotrimazole Mupirocin Nystatin Permethrin

ID=16855 This patient has a pruritic, erythematous rash affecting the feet. Microscopy of skin scrapings shows branching hyphae, confirming the diagnosis of tinea pedis (athlete's foot). Tinea pedis is a superficial infection of the epidermis caused by dermatophytes, especially Trichophyton rubrum and similar species. The organisms are widespread and are often acquired following exposure to public places while barefoot (eg, athletic facilities, swimming pools). They can also spread from other areas (eg, groin [tinea cruris], body [tinea corporis]) on the same patient via autoinoculation. Tinea pedis most commonly occurs between the toes (interdigital pattern) but in chronic cases can cause a hyperkeratotic rash extending up the sides of the feet (moccasin pattern). Tinea pedis is treated with a variety of topical antifungals, including azoles (eg, miconazole, clotrimazole), allylamines (eg, terbinafine), and tolnaftate. However, nystatin (primarily used for Candida infections) is not effective against dermatophytes (Choice D). (Choice A) Albendazole is an antihelminthic agent used to treat cutaneous larva migrans, which presents with pruritic, red/brown serpiginous tracks in the skin. The feet are typically the initial location of infection, but symptoms would not be confined to the interdigital skin. (Choice C) Topical mupirocin is used to treat impetigo, which presents with well-defined erythematous, erosive lesions with yellow crust. (Choice E) Permethrin is used for scabies, which can affect the interdigital skin but presents with small papules, pustules, and burrows. Microscopy of skin scrapings shows mites or ova and fecal matter. Educational objective: Tinea pedis is a superficial infection of the epidermis caused by dermatophytes (eg, Trichophyton rubrum). It most often presents as a pruritic, erythematous rash between the toes that frequently extends along the sole. Microscopy of skin scrapings shows branching hyphae. Treatment options include a variety of topical antifungals (eg, clotrimazole), but not nystatin.

A 35-year-old man comes to the office due to a rash and weight loss. He uses intravenous heroin. Examination shows tender anterior and posterior cervical lymphadenopathy. White plaques are present over the buccal mucosa and soft palate. Skin examination demonstrates a widespread rash consisting of lesions similar to that shown in the image below. ID=1761 Which of the following is the most likely cause of this patient's skin findings? Epstein-Barr virus Hepatitis C virus Herpes simplex virus type 1 Human herpesvirus type 8 Treponema pallidum

ID=1761 Oral thrush (eg, white buccal and palatal plaques) and bilateral tender lymphadenopathy in a patient with a history of intravenous drug use are highly suggestive of HIV infection. HIV is further supported by the presence of Kaposi sarcoma (KS), a vascular tumor common in HIV-positive individuals and an AIDS-defining malignancy. KS is strongly associated with human herpesvirus type 8 (HHV-8), as well as an impaired T-cell response (eg, HIV infection). HHV-8 infects vascular and lymphatic endothelial cells, causing their differentiation into a mixed phenotype thought to increase oncogenic potential. The viral genome also contains several oncogenes that cause endothelial proliferation (eg, inhibit cell cycle regulation and apoptosis), thus promoting tumorigenesis. In addition, HHV-8 induces production of angiogenic and inflammatory cytokines, creating a favorable environment for cell growth. AIDS-related KS most commonly involves the skin, presenting as multiple red, purple, or brown papules or plaques that appear on the lower extremities. Lesions can also develop on the mucosal membranes of the face and genitals, and frequently spread internally to involve the lungs and gastrointestinal tract (eg, bleeding/hemoptysis). Histopathology typically shows a proliferation of spindle-shaped endothelial cells with slit-like vascular spaces, red blood cell extravasation, and inflammation. (Choice A) Epstein-Barr virus is associated with Burkitt lymphoma, particularly in patients who are HIV positive. Burkitt lymphoma can involve the lymph nodes and may present with lymphadenopathy and constitutional symptoms (eg, weight loss). However, red-purple, papular skin lesions would not be expected. (Choice B) Hepatitis C virus can cause cryoglobulinemia, which often presents with palpable purpura (eg, nonblanching, hemorrhagic papules) on the lower extremities due to the deposition of immune complexes in small vessels. These lesions are typically associated with arthralgia and weakness. (Choice C) Herpes simplex virus type 1 can cause herpetic whitlow, a cutaneous finger infection characterized by clusters of vesicles (not red-purple papules) that may ulcerate and crust, often accompanied by burning and tingling. (Choice E) Treponema pallidum, which causes syphilis, can have variable cutaneous manifestations, depending on the stage of infection. Common skin lesions include chancres (eg, painless anogenital ulcers with indurated borders), diffuse erythematous macules, condylomata lata (eg, gray, wart-like growths), and gummata (eg, ulcerated nodules). Educational objective: Kaposi sarcoma is a vascular tumor strongly associated with human herpesvirus type 8. It typically presents as red, purple, or brown papules and plaques on the extremities of patients who are HIV positive.

A 23-year-old woman is brought to the emergency department due to one day of fever, vomiting, diarrhea, and muscle pains. The symptoms developed rapidly over the course of the day. The patient is confused and unable to answer questions. She has no known medical problems and takes no medications. She lives with her boyfriend. The patient's temperature is 39.1 C (102.4 F), blood pressure is 88/50 mm Hg, pulse is 120/min, and respirations are 20/min. A diffuse, erythematous, macular rash resembling a sunburn is noted on dermatologic examination; pelvic examination reveals a tampon in the vagina. Laboratory results are as follows: Leukocytes 8,000/mm3 Creatinine 1.4 mg/dL Aspartate aminotransferase (SGOT) 62 U/L Alanine aminotransferase (SGPT) 49 U/L A pregnancy test is negative. Activation of which of the following cells is most likely primarily responsible for this patient's condition? Basophils and macrophages Macrophages and T lymphocytes Mast cells and eosinophils Neutrophils and B lymphocytes Platelets and mast cells

ID=676 Fever, vomiting, diarrhea, muscle pains, and erythroderma (diffuse macular rash resembling a sunburn) are manifestations of toxic shock syndrome (TSS). TSS can rapidly progress to severe hypotension and multisystem dysfunction (eg, elevated creatinine, elevated transaminases). Desquamation, particularly on the palms and soles, can occur 1-2 weeks after the onset of illness. TSS has been associated with the use of tampons and nasal packing. Staphylococcus aureus strains producing toxic shock syndrome toxin (TSST) are responsible for most cases of TSS. TSST acts as a superantigen that activates large numbers of helper T cells (compared to a regular antigen, which activates few helper T cells). Superantigens interact with major histocompatibility complex molecules on antigen-presenting cells (eg, macrophages) and with the variable region of the T lymphocyte receptor to cause a nonspecific, widespread activation of T lymphocytes. Activation of T cells is responsible for the release of interleukin (IL)-2 from the T cells and IL-1 and tumor necrosis factor from macrophages. These ILs cause capillary leakage, circulatory collapse, hypotension, shock, fever, skin findings, and multiorgan failure. (Choice A) Basophils are not involved in TSS. Basophilia can be seen with some myeloproliferative disorders and, rarely, certain parasitic infections. (Choices C and E) Eosinophil activation is associated with reactions to allergens and parasites, hypersensitivity reactions to drugs, and some hematologic malignancies. Mast cells, when stimulated (due to antigen-induced cross-linking of IgE on their cell membrane), release histamine, prostaglandins, leukotrienes, and other inflammatory mediators; they are active in allergic reactions and anaphylaxis. (Choice D) B lymphocytes respond to activation by replicating and synthesizing antibodies specific for the antigen with which they were presented; they are not responsible for TSS. Neutrophilia is commonly seen with TSS. Educational objective: Superantigens (eg, toxic shock syndrome toxin) interact with major histocompatibility complex molecules on antigen-presenting cells and the variable region of the T lymphocyte receptor to cause nonspecific, widespread activation of T cells. This results in the release of interleukin (IL)-2 from the T cells and IL-1 and tumor necrosis factor from macrophages. This immune cascade is responsible for the manifestations of toxic shock syndrome.

A 6-month-old boy is brought to the emergency department due to poor feeding, irritability, and rash. He was well-appearing until 2 days ago, when he developed little interest in drinking and a progressive rash. Physical examination reveals diffuse erythema and desquamation that is most prominent at the neck, axillae, inguinal folds, and perioral region. The perioral area also has crusting and the lips are dry and cracked, but the mucosal membranes are normal. The epidermis easily comes off with gentle pressure at the erythematous areas. Which of the following is the most likely cause of this patient's symptoms? Autoantibody binding to epithelial cell surface Cell-mediated hypersensitivity Endotoxin-mediated inflammatory response Exotoxin-mediated skin damage Mast cell degranulation

ID=680 This patient's fever and desquamating rash are most consistent with staphylococcal scalded skin syndrome (SSSS), an infection caused by strains of exotoxin-producing Staphylococcus aureus. Exfoliative exotoxins act as proteases that cleave desmoglein in desmosomes, which are junctions that adhere epidermal cells together. Skin damage occurs as the exotoxins spread, causing separation of keratinocytes and superficial epidermal blistering. Gentle pressure causes detachment and shedding of the outer layer of skin (ie, Nikolsky sign). The rash, although diffusely erythematous and painful ("scalded"), is most prominent in the skin folds (eg, neck, axillae, groin). Perioral involvement (eg, erythema, crusting) is common, but the mucous membranes are spared. SSSS is most common in infants and young children, and skin findings are often preceded or accompanied by fever, irritability, malaise, and poor feeding. Treatment includes antistaphylococcal antibiotics (eg, nafcillin, oxacillin). (Choice A) Pemphigus vulgaris is caused by an autoantibody to desmoglein and results in a positive Nikolsky sign and epidermal erosions, blisters, and bullae. This disorder is uncommon in children, and mucous membranes are almost always involved. (Choice B) Cell-mediated (ie, type IV delayed-type) hypersensitivity, in which sensitized TH1 cells secrete cytokines to attract cytotoxic T cells and NK cells, is responsible for Stevens-Johnson syndrome and toxic epidermal necrolysis. Blistering, bullae, and Nikolsky sign may be seen, but mucosal involvement is almost always present. In addition, a preceding trigger (eg, medication, Mycoplasma pneumoniae infection) would be expected. (Choice C) Lipopolysaccharides (ie, endotoxin) released into the blood by gram-negative bacteria can cause a severe, widespread inflammatory response (septic shock). Purpuric lesions may occur with gram-negative sepsis, but Nikolsky sign would not be present. (Choice E) Crosslinking of IgE on the cell surface causes mast cell degranulation that is associated with allergic reactions. Urticaria (hives) appears as itchy, erythematous, blanching papules, not diffuse erythema or desquamation. Educational objective: Staphylococcal scalded skin syndrome occurs in infants and children due to the production of exfoliative exotoxins by Staphylococcus aureus. The toxins cleave desmoglein in desmosomes, leading to widespread epidermal blistering and shedding, especially with gentle pressure (ie, Nikolsky sign). Mucous membranes are spared.

An 18-year-old man comes to the office due to acne. The patient has had mild facial acne for the past 3 years that has responded well to topical agents. Since he started college 2 months ago, the acne has worsened and has spread to his torso and neck. Despite use of the topical agents, it has continued to worsen. The patient plays on the university's baseball team, which requires him to practice outdoors for several hours 3 days a week. He also swims in an indoor pool several hours a week. Since starting college, the patient has adopted a dairy-free, low-glycemic-index diet. Physical examination shows severe, nodulocystic acne on the face, neck, chest, and back. Which of the following most likely caused this patient's worsening acne? Dairy-free diet Exposure to chlorinated water Low-glycemic-index diet Methyltestosterone supplementation Ultraviolet radiation

ID=7585 This college athlete's rapidly worsening acne despite the previously effective use of topical agents raises concern for androgenic steroid supplementation. Androgens (eg, dehydroepiandrosterone, testosterone) are produced predominantly by the adrenal glands and gonads; they may also be supplemented exogenously. They bind to receptors in cytosol, translocate into the nucleus, and alter gene expression. In the skin, this results in increased sebum production by sebaceous glands and follicular epidermal hyperproliferation, which promote acne formation. The role that androgens play in acne pathogenesis is clearly demonstrated in the prepubertal and pubertal years, when a rise in adrenal and gonadal androgens coincides with acne formation. Hirsute women with pathologic hyperandrogenic states (eg, polycystic ovary syndrome) also frequently have coexisting acne. When acne suddenly becomes severe (eg, nodulocystic acne, acne fulminans), androgenic steroid supplementation should be considered as a possible cause, especially in athletes. (Choices A and C) Dairy consumption (particularly skim milk) and a high-glycemic-index diet are associated with acne. Natural hormones in milk and increased levels of insulin-like growth factor may play a role. (Choice B) Chlorine has antibacterial effects against Cutibacterium acnes, the bacteria which promotes inflammation in acne. Regular exposure to chlorinated water may improve acne. (Choice E) Ultraviolet radiation can cause sunburn or precipitate photodermatoses, such as the malar rash seen in systemic lupus erythematosus or drug-induced photosensitivity (eg, tetracyclines, sulfonamides). However, ultraviolet radiation would not suddenly cause severe acne. Educational objective: Androgens stimulate follicular epidermal hyperproliferation and excessive sebum production, thereby promoting acne development. Androgenic steroid supplementation is a known cause of acne, especially in competitive athletes.

A 2-year-old boy is brought to the office by his mother due to a rash that started 3 days ago. A similar red, itchy rash on the cheeks, trunk, and arms has occurred intermittently since infancy. The mother had thought that the rashes were related to the consumption of certain foods, but elimination of cow's milk and fruits did not improve the rash. The patient has had a few upper respiratory infections but no major illnesses. Vaccinations are up to date, and he takes no medications. He attends a day care facility. Vital signs are within normal limits. The rash on the patient's back is shown in the exhibit. Similar findings are observed on the cheeks and proximal upper extremities. The diaper area is clear, and no mucosal lesions are present. This patient's current condition increases his risk of developing which of the following additional disorders? ID=876 Celiac disease Asthma Langerhans cell histiocytosis Psoriatic arthritis Ulcerative colitis

ID=876 Atopic dermatitis (AD), or eczema, is a chronic inflammatory skin disorder common in children. The disease typically occurs at age <5, with many patients developing symptoms in infancy. AD typically presents as erythematous papules and plaques involving the face, scalp, trunk, and extensor surfaces. The diaper area is usually spared. In older children and adults, it often manifests as lichenification in a flexural distribution (eg, neck, wrists, antecubital and popliteal fossae). Eczema has a chronic course marked by exacerbations and remissions, and its severity tends to lessen with age. Intense pruritus is a hallmark of AD. AD can be associated with mutations affecting epidermal barrier proteins such as filaggrin, resulting in impaired skin barrier function. This increases immunologic exposure to environmental allergens and microbial antigens, leading to immune hypersensitivity. Affected patients usually have high serum IgE levels and peripheral eosinophilia. Children with AD often have a family history of atopy and are at risk of developing other atopic diseases, such as allergic rhinitis and asthma (allergic triad). (Choice A) Celiac disease is associated with dermatitis herpetiformis, which presents with pruritic vesicles and plaques on the buttocks and extensor surfaces of the extremities. In addition, this skin disorder is extremely uncommon in children. (Choice C) Langerhans cell histiocytosis is a dendritic cell disorder that can present in infancy or early childhood with a severe, refractory, erythematous or ulcerative rash on the scalp, trunk, or groin. This patient's rash flares and resolves without intervention. (Choice D) Psoriatic arthritis is associated with psoriasis. Manifestations of psoriasis include erythematous, well-demarcated papules and plaques with a silvery scale, which are not seen in this patient. (Choice E) Ulcerative colitis, a type of inflammatory bowel disease, can be associated with dermatologic manifestations, such as pyoderma gangrenosum (ulcerative rash) or erythema nodosum (painful raised red nodules). Both are uncommon in children, and neither is seen in this case. Educational objective: Atopic dermatitis (eczema) is a common, chronic inflammatory disorder caused by impairment of the skin's barrier function. Eczema presents with pruritus and erythematous papules and plaques and is associated with other atopic diseases, such as allergic rhinitis and asthma.

A 36-year-old man is admitted to the hospital with severe left leg pain after suffering a minor laceration to the leg 2 days earlier. Temperature is 39.5 C (103.1 F) and blood pressure is 85/45 mm Hg. On physical examination, the left leg is swollen and firm with erythema of the overlying skin. There is a small wound draining blood-tinged exudate on the left calf. The patient is taken to the operating room immediately, and a large area of necrotic tissue is excised. Gram stain of the necrotic tissue shows gram-positive cocci, and culture of the bacteria performed on a blood agar plate grows small colonies with a wide zone of surrounding hemolysis. Coagulase and catalase tests are negative, and pyrrolidonyl arylamidase (PYR) test is positive. Which of the following pathogens is most likely responsible for this patient's condition? Aeromonas hydrophila Clostridium perfringens Staphylococcus aureus Streptococcus agalactiae Streptococcus pyogenes

ID=8857 This patient has necrotizing fasciitis ("flesh-eating disease"), a severe soft tissue infection characterized by fulminant tissue necrosis and high mortality. The condition is often polymicrobial, but monomicrobial infection can occur with Streptococcus pyogenes, Staphylococcus aureus, and Clostridium perfringens. S pyogenes (group A Streptococcus) is a gram-positive cocci that is coagulase negative, catalase negative, and pyrrolidonyl arylamidase (PYR) positive. It also forms small colonies with a wide zone of beta-hemolysis and is sensitive to bacitracin, which has been widely replaced by the PYR test. S pyogenes can be transmitted through wounds and spreads rapidly to the deep layers of the skin and fascia due to production of hyaluronidase and streptokinase (converts plasminogen to plasmin). M protein is expressed on the surface and allows the bacteria to evade phagocytosis by preventing activation of the alternative complement pathway. The bacterium also secretes hemolysins O and S (cytotoxins that cause hemolysis) and pyrogenic exotoxins (superantigens that result in tissue injury and septic shock). Sudden-onset, severe pain and swelling are typically present at a site of trauma or recent surgery. Patients quickly become hypotensive and develop septic shock. Necrotizing fasciitis is treated with aggressive surgical debridement of all necrotic tissue plus empiric broad-spectrum antibiotics due to the high incidence of polymicrobial infection. (Choice A) A hydrophila is an oxidase-positive, non-lactose-fermenting, gram-negative rod. It can cause gastroenteritis, wound infections, and bacteremia following exposure to contaminated water. (Choice B) C perfringens is a large, spore-forming, anaerobic, gram-positive rod that is catalase and coagulase negative. It is implicated in food poisoning, clostridial myonecrosis (gas gangrene), and bacteremia. On blood agar, it produces a double zone of beta-hemolysis. (Choice C) S aureus is a gram-positive coccus that grows in clusters. It is catalase positive, coagulase positive, and PYR negative. S aureus produces large colonies and demonstrates beta-hemolysis. Like S pyogenes, S aureus can also cause monomicrobial necrotizing fasciitis . (Choice D) S agalactiae (group B Streptococcus) is a coagulase- and catalase-negative, PYR negative, gram-positive coccus that grows in chains. Colonies produce a narrow zone of beta-hemolysis that enhances when plated perpendicular to S aureus (positive CAMP test). S agalactiae can cause skin and soft tissue infections, as well as newborn sepsis and meningitis. Educational objective: Necrotizing fasciitis is a severe infection of the subcutaneous tissue and deep fascia and is a surgical emergency. The infection is often polymicrobial, but monomicrobial cases due to Streptococcus pyogenes (group A Streptococcus) can also occur. S pyogenes is a PYR-positive, beta-hemolytic, gram-positive coccus that grows in chains.

A 63-year-old woman comes to the office for a routine preventive examination. She has no significant medical problems and takes no medications. The patient consumes a balanced diet; gets regular exercise; and does not use tobacco, alcohol, or illicit drugs. She is up to date on breast, colon, cervical, and lipid screenings. Her physical examination findings are unremarkable. The patient expresses concern about wrinkles around her eyes that make her "look old." A decrease in which of the following is most likely responsible for this patient's complaint? Collagen cross-linking Collagen fibril production Collagenase synthesis Elastin degradation Proline hydroxylation

Multiple environmental factors, especially exposure to ultraviolet (UV) light, contribute to aging of the skin. UVB wavelengths are predominantly absorbed in the upper dermis and contribute to sunburn and increased risk of malignancy. UVA wavelengths penetrate deeper into skin and cause photoaging. UVA produces reactive oxygen species, which activate multiple inflammatory cell-surface receptors and nuclear transcription factors. This leads to decreased collagen fibril production, along with upregulation of matrix metalloproteinases (including collagenases) that subsequently degrade type I and III collagen and elastin (Choices C and D). Photoaging may be visible by age 30-35. Gradual thinning of the epidermis is seen, with reduction in subcutaneous fat, blood vessels, hair follicles, sweat ducts, and sebaceous glands. Rete ridges at the dermoepidermal junction become flattened. This loss of subcutaneous tissue causes the skin to become atrophic and more vulnerable to damage. In addition, there is increased crosslinking of collagen (Choice A), with deposition of collagen breakdown products. The atrophic dermis and increased collagen crosslinking, along with desiccation of the stratum corneum, produce the characteristic wrinkling of photoaged skin. (Choice E) Post-translational hydroxylation of proline residues contributes to the stability of the collagen triple helix. This reaction requires ascorbic acid (vitamin C); deficiency of vitamin C leads to scurvy, which is characterized by impaired production and reduced tensile strength of collagen. Educational objective: Photoaging is a product of excess exposure to ultraviolet A wavelengths and is characterized by epidermal atrophy with flattening of rete ridges. In addition, there is decreased collagen fibril production and increased degradation of collagen and elastin in the dermis.

A 31-year-old male farm worker comes to the physician complaining of an itchy rash on his chest. Physical examination reveals an annular and scaling plaque five centimeters in diameter with central clearing on the chest. KOH preparation of skin scrapings shows branching septate hyphae, and topical application of terbinafine was prescribed. Which of the following mechanisms of action explains the antifungal activity of this drug? Binding to ergosterol Inhibition of squalene epoxidase Blocking beta-D-glucan synthesis Preventing mitosis by binding tubulin Inhibition of fungal protein synthesis

The patient described in this vignette is presenting with symptoms of tinea corporis. Clinically tinea corporis presents as an annular scaling plaque with well-demarcated, raised erythematous borders and central clearing. Terbinafine is a commonly used drug for treatment of dermatophytosis. It belongs to the class of antifungals called allylamines and may be used topically or orally. Terbinafine inhibits synthesis of ergosterol of the fungal membrane by inhibiting the enzyme squalene epoxidase. Ergosterol is a cell membrane sterol that is unique to fungi and does not occur in human cell membranes. This drug tends to accumulate in skin and its appendages, and its side effects are mild by the topical route. (Choice A) Binding to ergosterol is a mechanism of action of polyene antifungals (amphotericin B and nystatin). (Choice C) Caspofungin belongs to the echinocandin class of antifungals. It blocks synthesis of β (1,3)-D-glucan, a main component of Candida and Aspergillus cell walls. (Choice D) Griseofulvin binds to polymerized microtubules and disrupts the fungal mitotic spindle, thus preventing fungal cell mitosis. It accumulates in the skin and its appendages and is only effective in dermatophytosis. (Choice E) Flucytosine is an antimetabolite antifungal that is transformed into 5-fluorouracil in the fungal cell. It inhibits fungal protein synthesis by replacing uracil with 5-fluorouracin in fungal mRNA. It is used in systemic fungal infections. Educational Objective: Terbinafine is used for treatment of dermatophytosis. It inhibits synthesis of fungal membrane ergosterol by suppressing the enzyme squalene epoxidase.

An 8-year-old boy is brought to the office due to a lower extremity skin rash for the past 3 days. The rash began as tender papules that quickly progressed to form vesicles and flaccid bullae that rupture easily. The patient has no mucosal lesions or other rashes and has never had similar symptoms before. He is up to date with recommended vaccinations and takes no medications. Skin examination findings are shown below. ID=15361 Which of the following is the most likely cause of this patient's skin findings? Autoantibodies targeted against hemidesmosomes Autoantibody-mediated epidermal cell separation Bacterial toxin-induced desmoglein 1 cleavage Defective epidermal barrier and T-cell dysregulation Inflammatory response to viral lytic infection

This child has a blistering skin rash with tan- to honey-colored crusts, consistent with bullous impetigo (BI). BI is a superficial infection by Staphylococcus aureus that is most common in young children. It is spread by direct contact, with the greatest incidence in warm, humid, crowded environments. Nonbullous impetigo is a similar syndrome with pustules and crusting but without significant blistering. Either form may occur in healthy skin (primary impetigo) or broken/inflamed skin (secondary impetigo). The blistering in BI is caused by production of exfoliative toxin A, a serine protease that targets desmoglein 1 in the superficial epidermis, by some strains of S aureus. Desmoglein is a cadherin component of desmosomes in epidermal cellular junctions; disruption by exfoliative toxin A causes a loss of cell adhesion and leads to formation of flaccid bullae. Exfoliative toxin A is also responsible for staphylococcal scalded skin syndrome, which presents with generalized erythema and flaccid bullae in flexural areas. (Choices A and B) Bullous pemphigoid is an autoimmune disorder associated with antibodies against hemidesmosomes and is characterized by tense, subepidermal blisters. Pemphigus vulgaris is characterized by intraepithelial cleavage and is due to autoantibodies against desmosomes (desmogleins 1 and 3); it presents with flaccid bullae, desquamation, and oral ulcerations. Both disorders cause scattered or widespread (rather than localized) lesions and are uncommon in children. (Choice D) Atopic dermatitis is associated with mutations in epidermal barrier proteins, leading to increased exposure to environmental antigens and immune hypersensitivity. It typically presents in infants and young children with erythematous papules and plaques involving the head, trunk, and extensor surfaces, and in older children and adults with lichenification in a flexural distribution (eg, neck, wrists, antecubital and popliteal fossae). (Choice E) Herpes simplex virus infection is characterized by recurrent clusters of mucocutaneous papules and vesicles that evolve into ulcerated and crusted lesions. It typically causes localized perioral or genital lesions rather than involving large regions on the extremities. Educational objective: Bullous impetigo is a superficial infection by Staphylococcus aureus that is most common in young children. It is characterized by a blistering skin rash with tan- to honey-colored crusts. The blistering in bullous impetigo is caused by exfoliative toxin A, which targets desmoglein 1 in epidermal cellular junctions and causes a loss of cell adhesion.

A 31-year-old woman comes to the physician with a velvety skin rash in her axilla, as shown in the image below. ID=936 Which of the following is the most likely cause of this patient's condition? Calcitonin hypersecretion Insulin resistance Serotonin hypersecretion Testosterone unresponsiveness Vitamin D resistance

This image shows the typical presentation of acanthosis nigricans (AN), which occurs most commonly in flexural areas. The most common locations are the axillae, posterior neck, and groin, but it may appear on other areas of the body such as the palms, soles, and oral mucosa & vermilion border. On physical examination, these lesions appear as hyperkeratotic, hyperpigmented plaques with a classic "velvety" texture. Skin tags (acrochordons) are pedunculated outgrowths of normal skin that are commonly present on regions affected by acanthosis nigricans. AN is associated with a number of diseases. Depending on the underlying condition, it is divided into benign and malignant forms. Benign AN is commonly associated with insulin resistance. Increased levels of insulin and insulin-like growth factors stimulate epidermal and dermal proliferation. Malignant AN is associated with underlying neoplasms, especially of the gastrointestinal and genitourinary tracts. The sudden appearance of such skin changes in middle-aged or elderly patients is suggestive of underlying malignancy. Gastric adenocarcinoma is the most common cause of malignant AN. (Choices A and E) Hypersecretion of calcitonin (as in medullary thyroid cancer) and vitamin D resistance are not associated with AN. (Choice C) Hypersecretion of serotonin can occur with carcinoid tumors, including the gastrointestinal variety. However, excess serotonin itself does not cause AN. (Choice D) Testosterone unresponsiveness (androgen insensitivity syndrome, testicular feminization syndrome) develops due to a defect in testosterone receptors in peripheral tissues. Such patients are phenotypic females who present with primary amenorrhea. The uterus is absent and the vagina ends as a blind pouch, but breast development is normal due to the production of estrogen by aromatase. Educational objective: Acanthosis nigricans presents with thickening and hyperpigmentation of skin in the flexural areas. The lesions have a classic "velvety" texture. Acanthosis nigricans is commonly associated with insulin-resistant states (eg, diabetes mellitus, acromegaly, obesity) and gastrointestinal malignancies.

A 79-year-old woman comes to the office due to bleeding from an ulcerative lesion on her right nipple that she first noticed several months ago. The lesion is itchy and is slowly increasing in size. Examination shows a crusty, thickened, erythematous, and ulcerated nodule with evidence of hyperpigmentation. Changes extend to the surrounding areola. Excisional biopsy reveals relatively well-circumscribed aggregates of large, polygonal, pleomorphic cells with abundant mitotic activity and surrounding necrosis. S-100 and HMB-45 stains are diffusely positive. Which of the following is the most likely diagnosis? Basal cell carcinoma Melanoma Paget disease of the breast Seborrheic keratosis Squamous cell skin cancer

This older patient has a crusted, ulcerated, hyperpigmented nodule on the nipple and areola. Biopsy was performed to differentiate Paget disease of the breast, an adenocarcinoma of the areolar epithelium, from nodular malignant melanoma. Histopathology revealed poorly differentiated cells with abundant mitotic activity and necrosis. Subsequent immunostaining was positive for S-100 (a protein expressed in cells derived from the neural crest such as melanocytes) and HMB-45 (a marker for immature melanosomes found in melanocytic tumors) indicating melanoma. Immunostaining is often used to assist the diagnosis of melanoma because histopathologic features are sometimes insufficient to confirm the diagnosis. Tumor cells that stain positive for multiple melanocyte markers (as in this case) are very likely to be melanoma. S-100 is positive in >95% of melanoma tumors, and HMB-45 is both highly sensitive (>95%) and highly specific (~100%) for the malignancy. (Choice A) Basal cell carcinoma usually appears on the face as a nodular, flesh colored papule. Although scaly trunk lesions can occasionally occur, this tumor is derived from basal epithelial cells and would not stain positive for melanocyte markers. (Choice C) Paget disease of the breast is a rare form of breast cancer that occurs due to intraductal spread of an underlying breast malignancy to the nipple skin. Although this can cause an itchy, ulcerative nipple lesion, biopsy would show malignant, intraepithelial adenocarcinoma cells (ie, Paget cells) within the epidermis that would not stain positive for melanocyte markers. (Choice D) Seborrheic keratosis is a benign skin lesion that is well demarcated (appears "stuck on") with a verrucous surface. These lesions are derived from immature keratinocytes and would not stain positive for multiple melanin markers. (Choice E) Squamous cell carcinoma is most common on the head and neck but can also occasionally arise on the trunk. It generally appears as a hyperkeratotic nodule, papule, or plaque. Because it is derived from epithelial cells (not melanocytes), multiple melanin markers would not be positive. Educational objective: Melanoma is often diagnosed when immunostaining of biopsy samples reveals multiple melanocyte markers (eg, S-100, HMB-45, MART-1).

A 53-year-old woman comes to the clinic due to a rash. For the past 3 weeks, she has had a pruritic rash of worsening severity involving the posterior thighs. The patient recently began an exercise program to lose weight and has been applying a topical analgesic cream to her thighs and buttocks after her workouts to relieve muscle aches. Medical history is unremarkable, and she does not use tobacco, alcohol, or recreational drugs. On physical examination, there is an erythematous rash with blisters, erosions, and weeping drainage involving the posterior thighs bilaterally, which is shown in the image below: ID=1115 Which of the following is the most likely finding on skin biopsy? Acanthosis Dyskeratosis Hyperganulosis Hyperkeratosis Spongiosis

This patient developed pruritic, erythematous blisters and erosions on the thighs following application of a topical cream. This presentation is consistent with acute allergic contact dermatitis (ACD), a type of eczematous dermatitis. Eczematous dermatitis is a class of skin conditions that vary in etiology but demonstrate similar clinical findings (eg, pruritic, erythematous, weeping papulovesicular rash) and are microscopically characterized by spongiosis. ACD is caused by a type IV (delayed) hypersensitivity reaction to an external antigen (eg, analgesic cream) on the skin surface, which incites an inflammatory response and causes pruritus and erythema at the site of contact. Increased vascular permeability in the dermis results in dermal edema, which can leak into the epidermis and accumulate in the intercellular spaces (ie, spongiosis). Microscopically, spongiosis appears as increased space between keratinocytes with prominent intercellular attachments (ie, desmosomes). Continued fluid accumulation can break the desmosomes, causing intraepidermal vesicles. With persistent antigen exposure, eczematous lesions may become less edematous and weepy over time and appear as raised, thickened (ie, lichenified), scaly plaques. Histology of chronic lesions typically shows thickening of the stratum spinosum (ie, acanthosis), stratum granulosum (ie, hypergranulosis), and stratum corneum (ie, hyperkeratosis) (Choices A, C, and D). (Choice B) Dyskeratosis is abnormal, premature keratinization of individual keratinocytes below the stratum granulosum. It may be seen in squamous cell carcinoma. Educational objective: Acute allergic contact dermatitis (ACD), caused by a type IV (delayed) hypersensitivity reaction to an antigen on the skin surface, typically presents with pruritic, erythematous, papulovesicular, weeping lesions. Microscopically, acute ACD is characterized by spongiosis (ie, accumulation of fluid between keratinocytes in the epidermis).

A 34-year-old man comes to the office due to a persistent skin rash. He has a 6-month history of a scaly rash involving the upper and lower extremities that is associated with mild pruritus. The patient has attempted treatment with topical antihistamines and emollients without relief. Medical history is notable for childhood asthma and orolabial herpes simplex virus infection. He does not use tobacco or alcohol and works as a sales associate for a home improvement store. On examination, the patient has skin lesions as shown in the image below. ID=1116 This patient's disorder is most likely associated with which of the following extradermal complications? Chronic glomerulonephritis Deforming joint disease Medullary thyroid cancer Peripheral neuropathy Pulmonary fibrosis

This patient with well-demarcated, scaly, erythematous plaques involving extensor surfaces of the extremities has typical features of psoriasis. Patients with psoriasis frequently develop additional complications, including: Psoriatic arthritis Nail changes: yellow-brown discoloration, pitting, thickening, or crumbling Inflammatory disorders of the eye: conjunctivitis, blepharitis, or uveitis Psoriatic arthritis is an inflammatory disorder affecting the synovium and often the insertion of tendons and ligaments (ie, enthesopathy). It typically manifest as asymmetric oligoarthritis or symmetric polyarthritis (resembling rheumatoid arthritis) that often affects the distal interphalangeal joints. In severe cases, patients can develop an aggressive and destructive arthritis mutilans. The risk is increased in patients who are human leukocyte antigen B27-positive. (Choice A) Chronic glomerulonephritis frequently occurs in systemic lupus erythematosus. However, lupus is typically associated with an erythematous malar rash. (Choice C) Medullary thyroid cancer is frequently linked to multiple endocrine neoplasia type 2. (Choice D) Skin lesions, renal insufficiency, and peripheral neuropathy (eg, mononeuropathy multiplex) are frequent findings in polyarteritis nodosa, a vasculitis affecting medium-sized arteries. (Choice E) Pulmonary fibrosis is a common extradermal manifestation of systemic sclerosis (scleroderma). Educational objective: Common complications of psoriasis include psoriatic arthritis, nail changes, and uveitis.

An 18-year-old man has chronic atopic dermatitis. He has been applying a corticosteroid cream daily to the flexural areas of his skin for several years, and the treated areas have been free of pruritus, papules, vesicles, and scales during that time. Medical history is notable for asthma and allergic rhinitis. Punch biopsy of the treated skin would most likely show which of the following findings? Adipose tissue hyperplasia Dermal atrophy Dermal perivascular lymphocytosis Epidermal hyperkeratosis Epidermal hyperplasia Epidermal intercellular edema

This patient's atopic dermatitis has been successfully treated with a topical corticosteroid, leading to suppression of the typical inflammatory features (eg, pruritis, papules, vesicles, scales). However, chronic daily use of topical corticosteroids can cause cutaneous atrophy due to the inhibition of epidermal keratinocyte proliferation and fibroblast function. Fibroblasts synthesize dermal collagens 1 and 3 as well as hyaluronic acid. When these components are diminished in the extracellular matrix of the dermis, indentations, striae, tightening, and thinning of the skin result, making dermal capillaries more visible. In addition, corticosteroids induce nitric oxide release, leading to capillary dilation (ie, telangiectasia). Corticosteroids can also cause degradation and extrusion of follicular epithelium, resulting in corticosteroid-induced acne. High-potency topical corticosteroids can cause adverse effects within 2-3 weeks of daily usage, with less potent corticosteroids taking longer to cause similar effects. Areas with thin skin (eg, flexural surfaces, intertriginous areas, face, genitalia) are more susceptible. Recovery can be expected within weeks to months after cessation of therapy. (Choice A) Chronic topical corticosteroid therapy can cause atrophy, not hyperplasia, of subcutaneous adipose tissue and muscle layers. (Choices C and F) Dermal perivascular lymphocytosis is characteristic of the initial stage of acute atopic dermatitis. Epidermal intercellular edema (ie, spongiosis) is also seen in acute atopic dermatitis and manifests clinically as small vesicles. However, given that the patient is asymptomatic with treatment, these findings would not be expected. (Choices D and E) Epidermal hyperkeratosis (ie, thickening of the stratum corneum) and epidermal hyperplasia (ie, increased cellularity in the stratum spinosum) are characteristic of chronic atopic dermatitis, which manifests with scales and lichenification. Neither finding is present in this patient. Educational objective: Local adverse effects of chronic topical corticosteroid therapy include cutaneous atrophy, telangiectasias, and corticosteroid-induced acne. Effects are more pronounced with high-potency corticosteroids and in areas of thin skin (eg, flexural areas).

A 37-year-old man comes to the office due to a rash. For the last several months, he has had numerous small lesions on the hands. The lesions are associated with occasional mild itching and bleed slightly if inadvertently struck against furniture or a wall. Examination findings are as shown in the exhibit. Which of the following is the most likely diagnosis? ID=17062 Atopic dermatitis Cutaneous warts Lichen planus Psoriasis Tinea infection

This patient has chronic, well-demarcated, erythematous plaques with a thick scale consistent with plaque psoriasis. Psoriasis is a common inflammatory skin disorder characterized by hyperkeratosis (overgrowth of the stratum corneum) and epidermal hyperplasia. It has a strong genetic predisposition and is often seen in patients with a family history of the disorder; other risk factors include smoking, heavy alcohol consumption, and obesity. Psoriasis has multiple variants (eg, guttate, pustular); however, up to 75% of cases are due to chronic plaque psoriasis. Plaques are well-defined and erythematous and may appear raised above the surrounding skin. They are typically covered in a coarse silver scale, which, when removed, results in pinpoint bleeding (Auspitz sign) of the underlying dilated capillaries. Plaques usually form in locations subject to trauma or friction (Köbner phenomenon); the extensor surfaces of the elbows and knees are most classic, but the dorsal surface of the hands, as in this patient, is also common. Other typical locations include the scalp, back, and gluteal cleft. They are often mildly pruritic but may be asymptomatic. (Choice A) Atopic dermatitis can present with erythematous, scaly plaques, but it is more common in flexural areas, shows less prominent scaling, and is usually associated with more prominent itching. (Choice B) Cutaneous warts (verruca vulgaris) are caused by human papillomavirus infection and present as discrete, hyperkeratotic papules. Although patients can have multiple warts, numerous scaly plaques are more consistent with psoriasis. (Choice C) Lichen planus presents with pink or purple plaques and papules. It is usually associated with severe pruritis and typically affects the volar surface of the wrists rather than the dorsum of the hands. (Choice E) Tinea manuum presents with scaly plaques on the hands. The lesions are typically very pruritic and often have an annular appearance with central clearing. Educational objective: Psoriasis is a common inflammatory skin disorder characterized by hyperkeratosis (overgrowth of the stratum corneum) and epidermal hyperplasia. Plaque psoriasis is the most common form and presents with chronic, well-demarcated, erythematous plaques with a thick, silver scale.

A 52-year-old woman comes to the office due to a 2-month history of oral lesions that cause pain with chewing and swallowing food. The patient did not seek treatment because she thought the lesions would go away on their own, but they have persisted. On examination, there are erosions of the buccal and gingival mucosa, as shown in the image below. There are several flaccid bullae with erosions scattered over her trunk. The blisters spread laterally with pressure, and traction on seemingly uninvolved skin produces blistering. Autoantibodies directed against which of the following structures are most likely responsible for this process? ID=2065 Image Basement membrane Connexin proteins Desmosomes Hemidesmosomes Tissue transglutaminase

This patient has pemphigus vulgaris (PV), an autoimmune bullous disease characterized by painful, flaccid bullae and erosions of the skin and mucosal membranes. Any mucosal site can be involved, but the oral mucosa is the most common. The bullae are structurally weak and often rupture prior to presentation, with only erosions remaining. The bullae spread laterally when pressure is applied on top (ie, Asboe-Hansen sign), and new blisters may form with gentle traction or rubbing (ie, Nikolsky sign). The underlying pathophysiology of PV involves autoantibodies directed against desmosomes, specifically desmogleins 1 and 3, which disrupt cohesion of keratinocytes. Biopsy of an active PV lesion will show intraepithelial cleavage with detached keratinocytes (acantholysis), retained keratinocytes along the basement membrane and an eosinophilic inflammatory infiltrate. Direct immunofluorescence of unaffected skin adjacent to PV lesions will show epidermal intercellular IgG and C3 deposits. Circulating autoantibodies against desmogleins are present in many patients and can aid diagnosis. (Choice A) Autoantibodies against cutaneous basement membrane proteins are characteristic of epidermolysis bullosa acquisita (forms tense acral bullae) and cicatricial pemphigoid (causes chronic conjunctivitis and scarring). (Choice B) No known cutaneous diseases result from connexin autoimmunity, but connexin defects do underlie some specific forms of palmoplantar keratoderma, which causes epidermal hyperproliferation of the palms and soles, or deafness-associated ichthyosis. (Choice D) Autoantibodies against hemidesmosomal proteins cause bullous pemphigoid, in which the bullae are usually tense and remain intact as the entire epidermis separates from the dermis (no intraepithelial cleavage). In addition, mucosal involvement is less common and Nikolsky sign is usually negative. (Choice E) Autoantibodies against tissue transglutaminase result in celiac disease and dermatitis herpetiformis, an autoimmune cutaneous eruption associated with gluten sensitivity. The eruption is characterized by grouped (herpetiform) erythematous papulovesicles that are intensely pruritic, not painful; oral mucosal involvement is rare. Educational objective: Pemphigus vulgaris is an autoimmune bullous disease characterized by autoantibodies directed against desmosomal proteins (eg, desmogleins). It presents with painful, flaccid bullae and erosions affecting the skin and mucosal membranes. The bullae spread laterally with pressure, and new blisters may form with gentle rubbing.

A 27-year-old man comes to the office for evaluation of a rash on both knees. The patient first noticed the rash 2 months ago. He has no other medical conditions and takes no medications. The patient smokes a half pack of cigarettes daily. Vital signs are normal. His knee is shown in the image below: ID=8569 Removal of the scale results in small bleeding points. As part of the treatment regimen, a topical vitamin D analog is prescribed. This medication is most likely to improve this patient's condition via which of the following mechanisms? Decreased activity of tyrosinase enzyme Decreased levels of irritating fatty acids Increased production of dermal cytokines Inhibition of dermal cyclooxygenase Inhibition of keratinocyte proliferation

This patient has psoriasis, a common skin disorder characterized by activation of T helper cells and proliferation of keratinocytes. Epidermal hyperplasia (acanthosis) produces erythematous plaques, and hyperkeratosis and confluent parakeratosis of the stratum corneum produce the characteristic scaling. The epidermal cell layer superficial to the dermal papillae may be thinned and contain dilated blood vessels, which can lead to pinpoint bleeding when the scale is removed (Auspitz sign). First-line treatment options for localized psoriasis include topical corticosteroids (eg, diflorasone) and vitamin D analogs (eg, calcipotriene, calcitriol). Vitamin D analogs activate the vitamin D receptor, a nuclear transcription factor, resulting in inhibition of T-cell and keratinocyte proliferation and stimulation of keratinocyte differentiation. Corticosteroids also have anti-inflammatory and antiproliferative properties; their mechanism of action is complementary to vitamin D analogs, and the 2 agents work well in combination. (Choice A) Tyrosinase is the primary enzyme responsible for oxidation of tyrosine, which serves as the rate-limiting step in synthesis of melanin. Tyrosinase inhibitors (eg, hydroquinone) decrease synthesis of melanin and can be used in treatment of hyperpigmentation disorders (eg, melasma). (Choice B) Acne vulgaris is promoted by proinflammatory fatty acid production by Cutibacterium (Propionibacterium) acnes. Benzoyl peroxide has bacteriostatic properties against C acnes and reduces production of these inflammatory fatty acids. (Choice C) Cutaneous warts are the most common manifestation of human papillomavirus infection. Imiquimod is a topical immunomodulator that likely induces local cytokine (eg, interferon-alpha, interleukin-6) production against the virus. (Choice D) Prostaglandin E2 (which is produced by cyclooxygenase-2) is involved in mediating the tumorigenic effects of ultraviolet light in the skin, promoting the formation of actinic keratosis (AK) and squamous cell carcinoma. Topical diclofenac, a nonsteroidal anti-inflammatory drug that inhibits cyclooxygenase, can be used to treat AK. Educational objective: First-line treatment options for localized psoriasis include high-potency topical corticosteroids and vitamin D analogs. Vitamin D analogs inhibit T-cell and keratinocyte proliferation and stimulate keratinocyte differentiation. Corticosteroids also have anti-inflammatory and antiproliferative properties; their mechanism of action is complementary to the vitamin D analogs.

A 35-year-old man comes to the office for evaluation of a rash on his elbows. The patient has had the rash intermittently for several months, but lately, it has become more persistent and bothersome. He also describes mild associated itching but no pain, and use of topical moisturizing lotion does not provide relief. The patient installs tile floors for a residential construction company. He has no chronic medical conditions and takes no regular medications. The patient does not use tobacco, alcohol, or recreational drugs. Vital signs are within normal limits. The rash on his elbow is shown in the image below, and a similar rash is present on the other elbow. ID=15408 Which of the following histologic changes are most likely to be found in this patient's skin lesions? Band-like lymphocytic inflammation Epidermal hyperplasia Granulomatous inflammation Intense eosinophilic infiltration Squamous cell dysplasia

This patient has scaly, erythematous plaques consistent with psoriasis. Psoriasis is most common in areas exposed to pressure or friction, such as the extensor surfaces of the elbows and knees, and minor trauma (eg, skin friction while installing tile floors) can often precipitate the formation of lesions (ie, Koebner phenomenon). In predisposed individuals, disruption of the epithelial barrier leads to activation of antigen-presenting dendritic cells and subsequently to a self-reinforcing inflammatory cascade characterized by recruitment and activation of T-helper cells and excessive proliferation of keratinocytes. Histologically, this results in epidermal hyperplasia (ie, acanthosis), often with elongated and clubbed rete ridges, corresponding to the typical erythematous plaques. There is parakeratosis (ie, retention of nuclei in the stratum corneum), which produces the characteristic silvery scales, as well as hypogranulosis (ie, decreased thickness of the granular layer). The epidermis superficial to the dermal papillae (ie, suprapapillary plate) is thin, and there are dilated capillaries in the dermal papillae, which results in pinpoint bleeding upon scale removal (ie, Auspitz sign). Clusters of neutrophils in the stratum corneum (ie, Munro microabscesses) and spinosum are also characteristic features. (Choice A) Lichen planus is characterized by band-like lymphocytic inflammation near the dermoepidermal junction as well as sawtooth-shaped rete ridges, hypergranulosis (ie, thickened granular layer), and hyperkeratosis. It typically presents with pruritic purple papules, frequently on the wrists and ankles. (Choice C) A granuloma is a nodular, inflammatory structure composed of aggregates of epithelioid macrophages and multinucleated giant cells. They are seen in a variety of infectious (eg, tuberculosis) and autoimmune (eg, sarcoidosis) diseases but are not seen in psoriasis. (Choice D) Eosinophilic infiltrates are a common finding in pemphigus vulgaris, bullous pemphigoid, eczematous skin diseases (eg, atopic eczema, allergic contact dermatitis), and dermatitis herpetiformis. These disorders generally present with vesicle or bulla formation, fragility of the skin, and often intense itching. Plaques and scaling, when present, are mainly due to persistent scratching. (Choice E) Actinic keratosis is characterized by squamous dysplasia and hyperkeratosis and is considered a premalignant lesion to squamous cell carcinoma. It is typically related to chronic ultraviolet exposure and commonly presents as a rough, scaly, erythematous lesion on sun-exposed areas (eg, face, dorsum of hands). Educational objective: Psoriasis is histologically characterized by diffuse epidermal hyperplasia with elongated and clubbed rete ridges (corresponding to the typical erythematous plaques), confluent parakeratosis (producing the characteristic silvery scales seen on examination), and dilated capillaries in the dermal papillae (pinpoint bleeding).

A 50-year-old woman comes to the office for evaluation of a mole on her back. The mole has been present for several years, but her daughter is concerned that it might have grown larger lately. The patient has no other medical conditions but had frequent sunburns during childhood. Dermoscopic examination of the lesion is shown in the image below. ID=16862 The lesion has multiple color variations, including dark brown, pink, red, and whitish areas. Excisional biopsy of the lesion is performed. The whitish areas (arrows) seen on histologic examination most likely represent which of the following underlying processes? Intense inflammation Melanocyte regression Proliferating, neoplastic melanocytes Proliferating, nonneoplastic melanocytes Vessel ectasia

This patient who had multiple sunburns as a child has a skin lesion with several classic features of melanoma, including Asymmetry, Border irregularity, Color variegation, Diameter ≥6 mm (hash marks), and Evolution over time. Color variegation is a common feature of melanoma; the different colors represent different areas of activity within the tumor: Red areas are due to vessel ectasia (dilation) and local inflammation (Choices A and E). Brown or black, flared areas along the border are due to advancing, neoplastic melanocytes (Choice C). White and gray areas appear when cytotoxic T lymphocytes recognize tumor antigens (eg, melan-A) and induce apoptosis, leading to malignant melanocyte regression (cleared patches). Melanocyte regression is also a hallmark of treatment response to melanoma immunotherapies, such as the programmed cell death receptor-1 (PD-1) inhibitor pembrolizumab. (Choice D) This patient's skin findings are characteristic of malignant melanoma. Proliferating nonneoplastic melanocytes do not generally play a role in the development or progression of melanoma. Educational objective: Melanoma lesions often have multiple color variations. The different colors represent different activities within the tumor. Whitish/gray areas occur when cytotoxic T cells recognize tumor antigens and destroy malignant cells, leading to melanocyte regression. Red areas arise due to vessel ectasia and local inflammation, whereas brown or black areas are generally due to advancing malignant melanocytes.

A 28-year-old woman is brought to the emergency department by her family due to acute left flank pain. The pain radiates to her groin and is associated with pink hematuria and urinary urgency. Past medical history is unremarkable, and the patient has never experienced similar symptoms. CT scan of the urinary system confirms the presence of a stone at the ureterovesical junction. The patient is given intravenous morphine sulfate for pain relief. Fifteen minutes later, she develops diffuse itching involving the arms, legs, and back. Which of the following is the most likely mechanism causing this patient's adverse reaction? IgE-dependent mast cell degranulation IgE-independent mast cell degranulation Inhibition of prostaglandin synthesis Precipitation of immune complexes T cell release of inflammatory cytokines

This patient with acute pruritus following administration of morphine most likely has medication-induced IgE-independent mast cell activation. A number of medications, including opioids, radiocontrast agents, and some antibiotics (eg, vancomycin), can induce mast cell degranulation by activation of protein kinase A and PI3 kinase. This results in release of several mediators, including histamine, bradykinin, heparin, and a number of enzymes and chemotactic factors. Common symptoms include diffuse itching and pain, bronchospasm, and localized swelling (urticaria). (Choice A) IgE-mediated degranulation is typically associated with environmental exposures such as foods or vespid stings or with medications such as beta-lactam and sulfonamide antibiotics. Opioids can cause IgE-dependent degranulation (very rare), but IgE-independent reactions are more common. (Choice C) Inhibition of cyclooxygenase 1 by aspirin and nonsteroidal anti-inflammatory drugs can trigger urticaria in patients with chronic idiopathic urticaria. These medications shunt arachidonic acid metabolites toward formation of leukotrienes, which increase vascular permeability. (Choice D) Serum sickness is a type III hypersensitivity reaction caused by precipitation of circulating immune complexes. Typical symptoms include fever, pruritic rash, and arthralgias starting 7-14 days after antigen exposure. Common triggers include beta-lactam and sulfonamide antibiotics. (Choice E) Acute allergic contact dermatitis is caused by a T cell-mediated type IV (delayed) hypersensitivity reaction to an antigen on the skin surface. Gross findings include erythematous, papulovesicular, weeping lesions. Educational objective: A number of medications, including opioids, radiocontrast agents, and some antibiotics (eg, vancomycin), can trigger IgE-independent mast cell degranulation. Common symptoms include diffuse itching and pain, bronchospasm, and localized swelling (urticaria).

A 25-year-old woman with newly diagnosed seizures comes to the emergency department with fever and skin rash. She had her first seizure 6 weeks ago, and an MRI of the brain revealed no structural lesions. She was started on phenytoin 4 weeks ago. Her temperature is 38.9 C (102 F). There is a diffuse confluent erythema involving 60% of the body, palpable generalized lymphadenopathy, and symmetrical facial swelling. Which of the following laboratory findings is most likely to be present in this patient? Antineutrophil cytoplasmic antibodies Blast cells with Auer rods Cryoglobulinemia Eosinophilia Microangiopathic hemolytic anemia

This patient's cutaneous and systemic findings after recently starting a new medication suggest drug reaction with eosinophilia and systemic symptoms (DRESS) syndrome. DRESS is a rare and potentially life-threatening drug reaction typically occurring 2-8 weeks after drug exposure. Commonly associated drugs include anticonvulsants (eg, phenytoin, carbamazepine), allopurinol, sulfonamides (eg, sulfasalazine), and antibiotics (eg, minocycline, vancomycin). Although the exact mechanism is unknown, it likely involves drug-induced herpesvirus reactivation followed by clonal expansion of T cells that cross-react with the drug. Patients typically develop fever, generalized lymphadenopathy, facial edema, and diffuse morbilliform skin rash that can progress to a confluent erythema with follicular accentuation. Other affected organs can include the liver (hepatomegaly, jaundice), kidney (acute interstitial nephritis), and lung (cough, dyspnea). Laboratory studies usually show eosinophilia, atypical lymphocytosis, and elevated serum alanine transaminase. Clinical findings improve over several weeks following withdrawal of the drug. (Choice A) Drug-induced antineutrophil cytoplasmic antibodies-associated vasculitis is most commonly linked to medications for hyperthyroidism (eg, propylthiouracil, methimazole) and hydralazine. Patients can develop constitutional symptoms, arthralgias/arthritis, and cutaneous vasculitis. (Choice B) Blast cells with Auer rods indicate acute myelogenous leukemia. Patients typically develop complications of pancytopenia, including fatigue (anemia), fever due to infection (neutropenia), and ecchymosis or epistaxis (thrombocytopenia). (Choice C) Mixed cryoglobulinemia is a small- to medium-vessel vasculitis caused by circulating immunoglobulin-complement complexes that precipitate on refrigeration. It is typically associated with chronic inflammatory states (eg, systemic lupus erythematosus, hepatitis C) and can present with systemic findings (eg, fatigue, arthralgias, myalgias) and palpable purpura in the lower extremities due to cutaneous vasculitis. (Choice E) Microangiopathic hemolytic anemia usually presents with anemia and fragmented red blood cells (schistocytes) on the peripheral smear. It has been associated with cancer chemotherapy drugs (eg, cisplatin, cyclophosphamide). Educational objective: Drug reaction with eosinophilia and systemic symptoms (DRESS) syndrome typically occurs 2-8 weeks after exposure to high-risk drugs such as anticonvulsants (eg, phenytoin, carbamazepine), allopurinol, sulfonamides (eg, sulfasalazine), and antibiotics (eg, minocycline, vancomycin). Patients typically develop fever, generalized lymphadenopathy, facial edema, diffuse skin rash, eosinophilia, and internal organ dysfunction.

A 64-year-old man comes to the physician with several lesions on his forehead. The patient has been a gardener for much of his life. The photograph below shows findings from the physical examination. ID=1993 On palpation, the lesions have a rough grainy texture. Biopsy reveals atypical keratinocytes with hyperkeratosis and parakeratosis. This patient's lesions put him at greatest risk for developing which of the following conditions? Basal cell carcinoma Dermatofibroma Kaposi sarcoma Melanoma Psoriasis Squamous cell carcinoma

This patient's forehead lesions are characteristic of actinic keratoses (AKs), which are small, scaly, erythematous lesions with a sandpaper texture occurring on sun-exposed areas. On light microscopy, AKs show hyperkeratosis (hyperplasia of the stratum corneum), parakeratosis (retention of nuclei in the stratum corneum), and atypical keratinocytes with pleomorphic nuclei and multiple mitoses. There may also be pigment irregularities and dilated blood vessels affecting the surrounding skin, which are consistent with chronic sun exposure. AKs do not invade the dermis and are considered to be premalignant lesions. Over several years, a small percentage of AKs transform into invasive squamous cell carcinoma (SCC), a change reflected by an increase in lesion size and thickness, dermal invasion, and metastatic potential. (Choice A) Basal cell carcinoma (BCC) is the most common skin cancer. It arises on sun-exposed areas and has a very low tendency to metastasize (in contrast to melanoma). BCC often presents as pearly papules with central depression or ulceration. Histologically, BCC features nests of basaloid cells and peripheral palisading of nuclei. It is far more likely for AK to progress to SCC than to BCC. (Choice B) Dermatofibromas (superficial benign fibrous histiocytomas) result from the benign proliferation of fibroblasts. They commonly arise on the lower extremities as solitary nodules. (Choice C) Kaposi sarcoma is associated with human herpesvirus-8 (HHV-8) and classically occurs in patients with HIV. Lesions typically appear as palpable macules, plaques, and nodules that are usually a dark brown to violet color. (Choice D) Melanoma is a cancer arising from the malignant transformation of melanocytes. Lesions have irregular borders and marked color variegation. AK is not a precursor to melanoma. (Choice E) Psoriasis commonly presents as hyperkeratotic plaques that affect the scalp, knees, and elbows. It is characterized histologically by clubbed rete ridges, hyperkeratosis, neutrophils in the stratum corneum, and perivascular lymphocytic infiltrates. Educational objective: Actinic keratoses (AKs) are small (usually <1 cm), erythematous epidermal lesions with adherent scale that are the result of chronic sun exposure. Histologic findings include keratinocyte atypia, hyperkeratosis, and parakeratosis. A small percentage of AKs progress to invasive squamous cell carcinoma; therefore, frequent monitoring is necessary.

A 43-year-old woman comes to the office due to acute back pain after dragging a heavy box. The pain is located in her lower back and radiates down the right posterior thigh to the foot. The patient describes the pain as "shooting" and grades it 8/10 in intensity. She has no bowel or bladder symptoms. The patient has tried over-the-counter analgesics with limited symptomatic relief. Vital signs are within normal limits. On physical examination, straight leg raise testing is positive on the right. Right hip extension is weaker when compared to the left. Patellar reflexes are 2+ and bilaterally symmetric, but the right Achilles reflex is absent. Which of the following nerve roots is most likely affected in this patient? L2 L3 L4 L5 S1

This patient's presentation is consistent with sciatica, a nonspecific term for low back pain that radiates down the leg. This condition occurs due to compression of the lumbosacral nerve roots and is most commonly caused by vertebral disc herniation or spinal foraminal stenosis (eg, due to degenerative arthritis of the spine). Irritation of the nerve roots results in characteristic dermatomal and myotomal deficits (radiculopathy) depending on the level of involvement. Patients may have worsening of their radicular pain when the symptomatic leg is extended at the knee and the hip is passively flexed by the examiner (straight leg raise test). The sciatic nerve is derived from the L4-S3 nerve roots and compression most often occurs at the level of L5 or S1. S1 radiculopathy is characterized by pain and sensory loss down the posterior thigh and calf to the lateral aspect of the foot. Patients may also have weakness on thigh extension (eg, due to denervation of the gluteus maximus), knee flexion (hamstrings), and foot plantarflexion (gastrocnemius) with an absent Achilles reflex. (Choices A and B) Patients with L2 or L3 radiculopathy typically have pain radiating down the anterior thigh with weakness on hip flexion (eg, due to denervation of the iliopsoas). Patellar and Achilles reflexes usually remain intact. (Choice C) L4 radiculopathy typically presents with pain/paresthesia radiating down the anterior thigh to the medial lower leg and foot. Patients can also have weakness on knee extension (eg, due to denervation of the quadriceps) with a diminished patellar reflex. (Choice D) Compression of the L5 root results in pain/paresthesia radiating down the lateral thigh and calf to the dorsal foot. Patients often have weakness on foot dorsiflexion and inversion (eg, due to denervation of the tibialis anterior), foot eversion (peroneus), and toe extension (extensor digitorum brevis). Educational objective: Sciatica is a nonspecific term for low back pain that radiates down the leg due to compression of the lumbosacral nerve roots (eg, from vertebral disc herniation). The S1 nerve root is commonly involved, resulting in pain/sensory loss down the posterior thigh and calf to the lateral aspect of the foot. Patients may also have weakness on thigh extension, knee flexion, and foot plantarflexion with an absent Achilles reflex.

A 34-year-old electric company worker comes to the physician with a skin rash on his right leg. He has not eaten any new foods or changed detergents, soaps, or lotions. On further questioning, the patient recalls that he recently worked on a repair job in an unmaintained, wooded area. He had atopic dermatitis as a child but no other significant illnesses. On physical examination, he appears uncomfortable and is constantly scratching his leg. His lungs are clear bilaterally and his heart sounds are normal. Examination of his right leg shows the findings in the image below. ID=1133 Which of the following cells is most responsible for causing the tissue damage seen in this patient? Basophils Eosinophils Mast cels Neutrophils Plasma cells T lymphocytes

This patient's pruritic skin rash following wilderness exposure is consistent with poison ivy dermatitis, a form of contact dermatitis. Poison ivy, poison oak, and poison sumac all produce urushiol, a small allergenic substance that causes an immune response when attached to proteins (ie, a hapten). Following contact with these plants, patients develop a highly pruritic, erythematous rash consisting of papules, vesicles, and bullae that may show signs of excoriation. The rash most frequently affects exposed skin (eg, legs, forearms) and often forms linear streaks as the patient walks past the plant, dragging it along the skin. Contact dermatitis is a type IV (delayed-type) hypersensitivity reaction that occurs in 2 distinct phases: The sensitization phase leads to the creation of hapten-specific T cells and takes 10-14 days. Cutaneous dendritic cells take up the haptens and express them on MHC-I and MHC-II molecules as hapten-conjugated peptides. These dendritic cells travel to the draining lymph nodes and interact with hapten-sensitive CD4+ and CD8+ T cells, causing activation and clonal expansion. The elicitation phase occurs within 2-3 days following re-exposure to the same antigen (or following sensitization after first exposure to a highly antigenic antigen such as urushiol). In this phase, the hapten is taken up by skin cells and causes activation of hapten-sensitized T cells in the dermis and epidermis. This results in an inflammatory response and the clinical manifestations of contact dermatitis. Depending on the etiologic agent, contact dermatitis can be mediated primarily by cytotoxic CD8+ T cells or CD4+ TH1 cells (that cause indirect damage by activating macrophages). In urushiol-induced contact dermatitis, CD8+ T cells are the primary effector cells and directly destroy keratinocytes expressing haptenated proteins. (Choices A and C) Mast cells and basophils, along with IgE, are primarily responsible for type I hypersensitivity allergic reactions. Mast cells play a role in modulating the response in contact dermatitis by affecting antigen presentation and T-cell recruitment and activation, but they are not the main effector cells in type IV hypersensitivity. (Choice B) Eosinophils are cells that play a role in the defense against parasitic organisms and allergic reactions. (Choice D) Neutrophils are the primary phagocytic killers of the innate immune system and do not play a significant role in type IV hypersensitivity reactions. They are more important in type III hypersensitivity reactions, in which deposited immune complexes activate complement and cause neutrophil-mediated tissue damage. (Choice E) Plasma cells are the principal cells responsible for the synthesis of immunoglobulins, which are directly responsible for type II and III hypersensitivity reactions. Educational objective: Poison ivy dermatitis is a form of allergic contact dermatitis, which is a type IV hypersensitivity reaction mediated primarily by T lymphocytes. It manifests as intensely pruritic erythematous papules, vesicles, or bullae that often form linear patterns.

A 27-year-old woman comes to the emergency department with a 1-day history of skin rash. Two weeks ago, she had an episode of recurrent genital lesions, which began with small papules and subsequently became vesicular with ulceration, crusting, and eventual healing. The patient has no other medical conditions and takes no medications. On physical examination, she appears comfortable. Temperature is 37 C (98.6 F). She has small linear erosions in the oral mucosa. Skin findings are shown in the image below. ID=11662 Which of the following is the most likely diagnosis? Disseminated herpes simplex infection Erythema multiforme Primary HIV infection Stevens-Johnson syndrome Varicella

This patient's rash is consistent with erythema multiforme (EM), an acute inflammatory disorder that can involve the skin of the extremities, face, trunk, and neck. Severe cases (EM major) can also affect oral mucous membranes and the tongue. The appearance of EM can vary, but patients typically develop erythematous, round papules that evolve into target lesions with a dusky central area, a dark red inflammatory zone surrounded by a pale ring, and an erythematous halo in the lesion's periphery. EM represents a cell-mediated immune process, with an inflammatory infiltrate predominated by cytotoxic CD8+ lymphocytes. EM is most frequently associated with infections (especially herpes simplex virus and mycoplasma) and may be due to an immune response against antigens deposited in the skin. EM can also be seen in association with certain medications (eg, sulfonamides), malignancy, and collagen vascular diseases. (Choice A) Disseminated herpes simplex infection is seen primarily in immunocompromised patients and presents with diffuse vesicles on an erythematous base. EM does not represent disseminated infection, but is a systemic immune response to a localized infection. (Choice C) Primary HIV infection can present with a macular or maculopapular rash associated with fever. The rash is usually well circumscribed and involves the trunk, face, and palms and soles. However, this patient's rash appears more typical of EM. (Choice D) Stevens-Johnson syndrome is characterized by rapid onset of erythematous macules with necrosis and skin sloughing. Systemic signs (eg, fever, hypotension) are common. Stevens-Johnson syndrome may also cause target lesions, but it is usually associated with medications (eg, sulfonamides, allopurinol, phenytoin). (Choice E) Varicella typically presents with fever and a vesicular eruption that can affect multiple areas, including the face, trunk, abdomen, and extremities. This patient's rash does not have the typical vesicular pattern seen in varicella. Educational objective: Erythema multiforme is a cell-mediated inflammatory disorder of the skin characterized by erythematous papules that evolve into target lesions. It is most commonly associated with herpes simplex virus.

A 24-year-old man comes to the physician with painful blisters on the shaft of his penis. The lesions erupted 2 days ago. The patient has had 5 lifetime sexual partners and is currently sexually active with one female partner; he uses condoms inconsistently. Examination shows multiple vesicular lesions on the penis, and Tzanck smear is positive for multinucleated giant cells. HIV testing is negative. The patient has had several similar episodes every year for the past 2 years but had been too embarrassed to seek treatment until now. Which of the following would most likely have prevented recurrence of this patient's condition? Continuous daily valacyclovir Immunoglobulin during the first episode Lamivudine with recurrence of blisters Regular condom use after the first episode Weeklong course of acyclovir during the first episode

This patient's recurrent vesicular lesions and positive Tzanck smear are consistent with recurrent genital herpes simplex virus (HSV), generally due to reactivation of latent HSV-2 infection within the S2, S3, and S4 dorsal root (sensory) ganglia. Recurrence of genital HSV can be suppressed or minimized with daily oral valacyclovir (preferred as it is dosed once daily and has good bioavailability), acyclovir, or famciclovir. Although the antiviral agents may not be active against latent virus forms, they can suppress further multiplication as soon as reactivation occurs. Suppressive therapy would likely have been offered to this patient with multiple yearly episodes of recurrence; it is generally continued for years with periodic re-evaluation. (Choice B) Immunoglobulin therapy can be used for varicella postexposure prophylaxis (eg, in neonates or pregnant women) but is not commonly used for genital HSV infections. (Choice C) Lamivudine is a nucleoside analog reverse transcriptase inhibitor with activity against HIV and hepatitis B virus. It is not known to be effective against HSV-2. (Choice D) Although condom use may prevent sexual transmission of HSV-2 (and therefore might have prevented this patient from acquiring his primary genital HSV-2 infection), it does not reduce the frequency of recurrence of genital herpes. (Choice E) A short (7-10 days) course of treatment with oral acyclovir during primary genital HSV infection usually reduces the duration of viral shedding, time for lesional healing, and local pain; however, it does not appear to alter recurrence rates. Educational objective: A new-onset genital vesicular rash with a positive Tzanck smear in a previously asymptomatic patient is suggestive of primary genital herpes simplex virus (HSV) infection due to HSV-2. Recurrences of genital herpes can be reduced through daily treatment with oral valacyclovir, acyclovir, or famciclovir. Condom use can help prevent a primary genital HSV infection but does not prevent reactivation of latent infection.

A 22-year-old woman comes to the office due to several years of persistent facial blemishing that has failed to respond to over-the-counter treatment. The patient has no other medical conditions. She does not use tobacco, alcohol, or illicit drugs. Skin examination findings are shown in the image below: ID=15379 Which of the following most likely contributed to the pathogenesis of this patient's skin condition? Androgen-induced involution of sebaceous glands Bacterial degradation of apocrine gland secretions Estrogen-stimulated secretory function of sebaceous glands Increased desquamation of follicular epithelial cells Proliferation of lipid-utilizing bacteria within pilosebaceous follicles

This patient's red papules and pustules on the face are consistent with inflammatory acne. The pathogenesis of acne involves the following: Hyperkeratinization due to abnormal epithelial growth and differentiation of corneocytes leads to keratin plug formation in the pilosebaceous follicles. These blocked follicles are referred to as comedones (ie, whiteheads and blackheads). In response to androgen stimulation (eg, during pubertal adrenarche), sebaceous glands enlarge (not involute) and increase production of sebum, a lipid-rich substance that facilitates obstruction of pilosebaceous follicles (Choice A). Cutibacterium acnes, an anaerobic bacteria that relies on sebum as a nutrient source, proliferates in occluded follicles, triggering an inflammatory response that results in the red papules and pustules characteristic of nodulocystic acne. (Choice B) Bacterial metabolism of apocrine secretions contributes to body odor. Bromhidrosis is a condition characterized by excessive and offensive body odor. (Choice C) Estrogen has antiandrogenic effects and inhibits sebum production. For this reason, estrogen-containing oral contraceptives help to improve inflammatory acne. (Choice D) As epithelial stem cells undergo mitosis in the basal layers of the epidermis, newer cells form and undergo differentiation as they migrate toward the surface, replacing epithelial cells that desquamate. Increased epithelial mitosis and desquamation (ie, topical retinoids) helps extrude debris from comedones, limiting acne formation. Educational objective: Acne is characterized by the obstruction of pilosebaceous follicles (ie, comedones) due to hyperkeratinization and excessive sebum accumulation. Androgens stimulate production of sebum, which serves as a nutrient source for Cutibacterium acnes. Within the follicles, C acnes proliferation triggers an inflammatory response, resulting in formation of red papules and pustules.

A 26-year-old man is being screened as a potential kidney donor for his mother, who has end-stage renal disease. The patient has no history of diabetes, hypertension, or kidney disease and is up to date with all recommended vaccinations. Physical examination is unremarkable. Blood type and crossmatch testing demonstrate biological compatibility. As part of the evaluation, screening for transmissible diseases is performed and is notable for the following results: HIV-1 antibody negative Hepatitis B core antibody, IgM negative Hepatitis B core antibody, IgG negative Hepatitis B surface antigen negative Hepatitis B surface antibody positive Hepatitis C virus antibody negative Cytomegalovirus antibody, IgM negative Cytomegalovirus antibody, IgG positive Based on these data, the patient most likely has had which of the following conditions in the past? Esophageal ulcer Hepatitis Mononucleosis-like illness Retinitis Vesicular skin rash Viral pneumonia

This patient, who is HIV negative, has a positive titer for cytomegalovirus (CMV) IgG, suggesting prior CMV infection. CMV is a rare cause of disease in the immunocompetent, with the virus more typically responsible for subclinical infection. When primary CMV infection does result in clinically evident illness, affected individuals appear to have a systemic mononucleosis-like syndrome characterized by fever, malaise, myalgia, atypical lymphocytosis, and elevated liver transaminases. In contrast to Epstein-Barr virus mononucleosis (most common), heterophile antibodies are not usually present in patients with CMV mononucleosis (negative Monospot test). (Choices A and D) Odynophagia (painful swallowing) associated with linear esophageal ulcers due to CMV infection is seen in patients with HIV. Retinitis is one of the most common manifestations of CMV infection in patients with HIV. (Choice B) Disseminated CMV can cause diarrhea and hepatitis in the immunocompromised. In this patient, the combination of a positive hepatitis B surface antibody with a negative hepatitis B surface antigen and a negative hepatitis B core antibody reflects immunization against hepatitis B, not prior hepatitis B infection. (Choice E) A vesicular skin rash is seen in association with herpes simplex or herpes zoster virus infection. (Choice F) CMV pneumonitis typically occurs after organ transplantation. Educational objective: Cytomegalovirus (CMV) is typically associated with subclinical infection in the immunocompetent, with the occasional individual developing a mononucleosis-like syndrome that is heterophile antibody (Monospot) negative. In the immunocompromised, primary or reactivated CMV infection can result in severe retinitis, pneumonia, esophagitis, colitis, or hepatitis.

An 8-year-old girl is brought to the office for evaluation of a skin rash on her upper back. The rash is not painful but is mildly pruritic. She has no prior medical conditions, takes no medications, and is up to date with vaccinations. Vital signs are within normal limits. Right scapular area skin examination is shown in the exhibit. The remainder of the examination is normal. Which of the following is the most likely cause of this patient's current condition? ID=15579 Deep tissue invasion by normal skin flora Epidermal damage from excessive sun exposure Immunologic response to bacterial pharyngitis Microbial infection of keratinized structures Proliferation of pathogen transmitted by tick bite

This patient, with a mildly pruritic, polycyclic rash with a raised, scaly border and central clearing, has tinea corporis. Patches of tinea corporis are typically round or ovoid but may become confluent to form a "flower petal" shape. Skin contact, especially in warm and humid environments, is a common risk factor, and patients often have concurrent infections in other body areas. Any species of dermatophyte may cause this condition, but Trichophyton rubrum is the most frequent culprit. Dermatophyte infections (eg, tinea corporis, tinea pedis, tinea cruris) infect keratinized matter in the stratum corneum of the superficial epidermis but do not invade the dermis and subcutaneous tissues. The diagnosis is confirmed with potassium hydroxide (KOH) preparation of skin scrapings, which can show the characteristic segmented hyphae and arthrospores. (Choice A) Cutaneous candidiasis (eg, intertrigo) is an inflammatory condition involving approximated skin surfaces such as the axillae, groin, inframammary folds, or abdominal folds. Candida albicans is a commensal organism that is normally present on the skin and can become invasive if there is disruption in the normal skin barrier. (Choice B) Sunburn presents with erythema, itching, and pain in sun-exposed areas. Sunburned skin may undergo superficial desquamation (peeling), but this patient's rounded lesions with peripheral scaling are more consistent with tinea corporis. (Choice C) Patients with untreated streptococcal pharyngitis can develop erythema marginatum, a fleeting pink or red rash with central clearing that can appear, disappear, and reappear within hours. This is a hallmark feature of rheumatic fever and is usually accompanied by other signs such as migratory arthritis, carditis, subcutaneous nodules, and Sydenham chorea. (Choice E) Acute Lyme disease is characterized by flu-like symptoms and erythema migrans, a slowly spreading erythematous rash with central clearing at the site of the tick bite. Disseminated disease can present with multiple lesions, but the lesions are flat or only slightly raised and the border is indistinct rather than scaly; they often develop a targetoid appearance. Educational objective: Tinea corporis presents with round or ovoid lesions with a raised, scaly border and central clearing. Trichophyton rubrum is the most common cause and infects keratinized matter in the stratum corneum of the superficial epidermis but does not invade the dermis and subcutaneous tissues.


Related study sets

Unit 2 Egypt: Section 2 The Old Kingdom

View Set

CSH081 - Ultrasound in General Radiography, PET/CT/MRI, Radiographer Commenting, Patient Advocacy

View Set